Supply Final 2

Ace your homework & exams now with Quizwiz!

An advantage of "early supplier involvement" is that it:

1. Generally leads to higher-quality products. 2. Decreases time to launch new products. 3. Avoids supply problems once the product is launched.

Which of the following statements are reasons why operations management is important?

1. Operations management is responsible for much of the value created by organizations. 2. Operations management is a key source of competitive differentiation among firms. 3. Efficient and productive operations drive the economic well-being of nations.

Growth of the supply chain management perspective in operations management results from the advent of:

1. Technology and infrastructure advances. 2. A focus on core capabilities. 3. Collaborative networks.

A packaging plant is running a filling process where packages are filled with 24 oz. +/- 0.08 oz. A capability study reveals that the process mean is 24 oz. with a standard deviation of 0.02 oz. What is the capability of the packaging line?

1.33

The specifications for the diameter of a molded part are 10 mm ± 0.5 mm. The actual average and standard deviation from 250 parts sampled is 10.1 mm and 0.1 mm, respectively. What is Cpk?

1.34

What is the takt time in minutes, if 500 units are needed per day? The company works 2, 8-hour shifts per day.

1.9 minutes

The following sequence shows four operations for a computer chip assembly process and the effective capacity of each. [List of steps and rates] Suppose the utilization is 70% of effective capacity. What is the actual output of the process?

140 chips/hour (200*.7)

Jones Company uses widgets as component parts. In a particular week it has gross requirements for 250 units of widgets, beginning inventory of 50 widgets, and scheduled receipts of 350 widgets. Available inventory for the next period is:

150 Beginning inventory (50) + scheduled receipts (350) - gross requirements (250) =150

Next year a tire company wants to have an inventory turnover rate of 22 times per year. To achieve this turnover rate, what should be the average number of days of supply?

16.6 days

Fifteen samples of size 10 are taken from a stable process. The average of the sample means is 32.8, and the average range of the samples is 1.5. What is the upper control limit for the R chart? Use the information contained in the attached chart. Picture

2.666

What is the take time in seconds if 1,000 units are needed per day? The company works one 8- hour shift?

29 Seconds

The most important stage in the negotiation process is: A) Planning for the negotiation. B) The face-to-face meeting. C) When the contract is signed. D) Forming the negotiating team.

A

Batch process

A process in which goods or services are produced in groups (batches) and not in a continuous stream - Moderate volume, variety (Bakery, automotive parts, cinema)

Which process activity is due to other factors (e.g., insufficient operating capacity, lack of material, equipment breakdowns)? A) Delay B) Operations C) Storage D) Inspection

A) Delay

Jones Company has a make-to-order orientation. It most likely does NOT use:

A. Continuous process structure

Suppose you learn that a process has a very long waiting time. Which of the following would decrease the waiting time?

A. Decrease utilization

A relationship with a supplier that is characterized by high levels of distrust, little communication, and short-term transactions is called a(n):

Adversial Relationship

Logistics customer service benefits include: Availability B) Lead-time performance C) Service reliability D) All of the above E) None of the above

All of the above

Process thinking causes managers to address critical process elements, including:

All of these (Activities, inputs/outputs, Flows, structure, resources, and metrics)

Which of the following is NOT true about "order qualifiers"?

All of these statements are true: These are product traits that must meet a certain level in order for the product to even be considered by customers. The firm must perform acceptably on these traits, usually at least as well as competitors' offerings. Customers may not be aware of any level of performance in excess of those minimum levels that they have established.

"Quality management" is:

An approach that develops culture and tools with a focus on quality

Service blueprinting

An approach to similar to process mapping that analyzes the interface between customers and service processes - customer actions, front-office contact employee actions, back-office contact employee actions, support processes, physical evidence

Which of the following products would be most likely "make to stock"?

An automobile

When should the metrics used in evaluating a process be developed? As soon as we have identified the value proposition After we have identified the critical process When we have done the "as is" analysis of the process After we have completed the future state analysis of the process

As soon as we have identified the value proposition

What might a company do to reduce the amount of cycle stock it holds?

Attempt to reduce order costs

The identification, acquisition, positioning, and management of resources and capabilities that a firm needs to attain its strategic objectives is: A) Sourcing. B) Supply management. C) Supply chain management. D) Spend analysis.

B

Which of the following statements that might be made by a business executive best expresses the overall philosophy of Total Quality Management? A) "We've achieved Six Sigma. That's good enough." B) "There's always room for improvement." C) "The workers need to work harder to prevent quality problems." D) "We never sell defective products to customers."

B) "There's always room for improvement."

The 'arrow' symbol indicates what type of activity in a process? A) Operation B) Transportation C) Delay D) Storage

B) Transportation

The two types of data involved in quality control are: A) Assignable factors and random factors B) Variables and attributes C) Random measurements and 100 percent measurements D) Accuracy and precision

B) Variables and attributes

Which of the following is NOT considered in the service process matrix? A) Customization B) Volume C) Customer D) Labor intensity

B) Volume

One implication of increased utilization in a process is that: A) Process cost typically will go up B) Waiting time typically will go up C) Variance typically will be reduced D) All of these

B) Waiting time typically will go up

You are sitting next to a person in business class on a flight from Los Angeles to Sydney, Australia. You mention to that person that you got your ticket two months ago for only $12,500. The person responds that she bought her ticket two days ago for $7,800. This sometimes happens because airlines often use an approach called: A) Capacity management B) Yield management C) Load management D) Workforce leveling

B) Yield management

You walk into a hair stylist shop. All stylists are busy and you sit in the waiting area. You are in which of the following process activities? A. Operation B. Delay C. Storage D. You are not in a process activity

B. Delay

Make/buy/subcontract is typically considered at what time horizon when doing capacity planning?

B. Medium-term

A bottleneck activity in a process is generally the activity with:

B. The least capacity

What is the primary objective of line balancing?

B. To match output rates with actual demand

Which of the following is NOT considered in the service process matrix?

B. Volume

A detailed description of an "end item" along with a list of all of its raw materials, parts, and subassemblies is shown in a:

BOM.

A restaurant chain that assembles meals at a centralized facility is using:

Back-office processes

cross-functional

Because of its complexity, insourcing/outsourcing analysis should be done by a ___ ___ team.

A company has determined the ideal number of locations by looking at the trade-offs described in the text. It decides to open more distribution centers than this "ideal." This decision would most likely result in ________ and ________.

Better service to customers and higher cost

ISO 9000 defines a set of internationally accepted standards for:

Business quality management.

The capability to resist and recover from supply chain disruptions is: A) Supply chain risk. B) Total cost of ownership. C) Supply chain resilience. D) Sustainability.

C

Under Juran's Law, whenever a problem occurs, what percentage of the time is the problem the result of a system/process error? A) 15% B) 50% C) 85% D) 100%

C) 85%

"Zero defects" is a concept of quality management of which thought leader? A) Deming B) Juran C) Crosby D) Imai

C) Crosby

An automated process can be compared to a more labor-intensive process using: A) A service blueprint B) Line balancing C) Indifference analysis D) Mass customization

C) Indifference analysis

If a tangible good or service performs the promised function dependably, we say that it has high: A) Conformance B) Durability C) Reliability D) Perceived quality

C) Reliability

Which of the following is NOT a basic activity type in processes? A) Delay B) Operation C) Reporting D) Storage

C) Reporting

Under Juran's Law, whenever a problem occurs, what percentage of the time is the problem the result of a system/process error?

C. 85 percent

"Zero defects" is a concept of quality management of which thought leader? A. Deming B. Juran C. Crosby D. Imai

C. Crosby

Using technology, such as self-checkouts in grocery stores, to enable customers to complete the service delivery and transaction themselves is most commonly used for:

C. Mass services

Kaizen Events are most appropriate for which type of operations problem?

C. Process-based

An online auction used for sourcing:

Can hurt supplier relationships.

Taxes and insurance costs are an example of which of the following costs?

Carrying (or holding) costs

Significant changes in an MPS quantity several weeks into the planning horizon can:

Cause inconsistencies in the MRP.

You are given the following data for taillight assemblies: Picture Which of the following tools does this data represent?

Check sheet

What is a check sheet?

Check sheets collect, organize, and display data to reveal patterns.

A trucking company which provides shipping services to the public is a:

Common Carrier

A trucking company which provides shipping services to the public is a:

Common carrier.

Continual improvement of processes is important because:

Competitors are improving their processes, customer preferences change over time, technology is constantly changing

Which of the following statements about projects and project management is NOT true?

Completing a project within budget is the most critical criterion for project success.

The stage of a product/process innovation in which managers determine the target values of product attributes, volume, and price is:

Concept Development

Scantron Inc. claims that its competitors have to recall 10 percent of their products to fix defects, while it only has to recall 5 percent. Scantron is emphasizing which dimension of quality?

Conformance

Jones Company has a make-to-order orientation. It most likely does NOT use:

Continuous process structure

In a Cost of Quality analysis, "failure costs" refers to:

Costs associated with defects

Value stream mapping A) Is a simplified version of process mapping B) Uses the same terminology and symbols used in typical process mapping C) Is another name for a future state map D) Is more comprehensive than typical process mapping

D) Is more comprehensive than typical process mapping

If a company can eliminate all sources of variance in a process: A) It will be able to operate at maximum capacity at all times B) There will be no constraints in the process C) Continuous improvement will not be necessary D) None of these are true

D) None of these are true

If a company can eliminate all sources of variance in a process:

D. None of these are true

The types of costs included in a cost of quality analysis include: A. Production, delivery, and marketing. B. Internal and external. C. Direct and indirect. D. Prevention, appraisal, and failure.

D. Prevention, appraisal, and failure.

Zanadu Corp. has dedicated equipment and workers in a regularly occurring sequence of activities. Zanadu has which type of operations layout?

D. Product layout

An application of the DMAIC process to product design is known as:

Design for Six Sigma.

Which of the following is NOT a role of logistics management?

Determining the number and locations of warehouses.

Recycling is a major consideration during which phase of the product life cycle?

Disposal

Downcycling, as described in the chapter, is a critical consideration during which stage of the product life cycle? Usage Disposal Packaging/transportation Extraction

Disposal Of the four areas, while the issues of downcycling are encountered in all four, this issue is most directly relevant during the last stage, disposal.

The center-of gravity method of determining a facility location:

Does not require that qualitative factors be considered in location decisions.

Benefits of ERP systems include all of the following EXCEPT:

Easy implementation.

An example of an integrative technology is a/an:

Enterprise resource planning (ERP) system

___________ inventory is the costliest form of inventory one can hold.

Finished goods

New home construction uses which type of layout?

Fixed-position

John Jones, CEO of Joes Corp., is unhappy because each product his company makes takes a unique route through the facility, so processing times tend to be high. His company most likely has a:

Functional Layout

Which of the following is NOT a stakeholder, as considered from the P-People perspective of sustainability? Government Customers Investors Workers

Government

One function of distribution centers (DC) is "consolidation." This means that the warehouse/distribution center is used to:

Group small inbound loads into larger outbound loads.

All of the following can increase supply chain risk except:

Holding higher levels of inventory of critical materials.

The Smith Company has decided to change its shipping mode from truck to rail. Its inventory levels will:

Increase.

Random Corp. is a company which offers logistics services such as storage, transportation and information processing to its customers. Companies like Random Corp. are called Public warehouse with benefits Contract warehouses Common Carriers Integrated Service Providers

Integrated service providers

The connection of products and machines for sharing data with other devices is

IoT

Which of the following operations management systems/approaches is most appropriate when dealing with environmental sustainability?

Lean Systems

For which market orientation is accurately forecasting the amount of finished goods needed likely to be the most important for a firm's financial performance?

Make to stock

Which of the following are not considered a relevant aggregate planning cost?

Management and supervisory training cost

The TQM view of organizational structure states that

Management must support employees.

The TQM view of organizational structure states that:

Management must support employees.

Order-to-delivery lead time generally would be longest for which type of product?

One that is engineered to order

Which of the following is NOT an input to an MRP system?

Order releases.

Fairtrade deals with the following elements of the Triple Bottom Line?

People.

Within the supply chain and operations management system, the value proposition is most effectively communicated to the rest of the organization through which of the following mechanisms?

Performance measures

Materials requirements planning (MRP) systems are used primarily to:

Plan orders.

In line-balancing, the relationships between tasks in a process is shown in a:

Precedence diagram

______ are companies that own and operate transportation equipment to transport their own products.

Private carriers

Which of the following would benefit the most from MRP?

Production of vacuum cleaners.

A law firm is an example of a:

Professional Service

Which of the following is NOT a core value of TQM?

Quality assurance managers should have the most power in the organization.

Capacity requirements planning does all of the following EXCEPT:

Recalculate the MRP plan

Jones Company decided to limit deliveries to selected areas to specific days of the week to save money on transportation. This is an example of:

Scheduled delivery consolidation.

Logistics is responsible for all of the following EXCEPT:

Setting product quality standards.

Little's Law suggests that the key to increased throughput is: Shorter flow times Fewer defects Eliminating waste Increased inspection

Shorter flow times

Little's Law suggests that the key to increased throughput is:

Shorter flow times.

Phil Bord is a CEO for a large auto manufacturer and is interested in improving the product quality. Phil had overheard his friend mention Six Sigma. Considering that Phil has no idea what Six Sigma is, what are some key points to help Phil out?

Six sigma quality is the result of a well-defined and structured process.

The identification, evaluation, selection, and management of suppliers is:

Sourcing.

supplier scorecard

Suppliers receive regular feedback, typically monthly or quarterly, in a ___ ___.

Determining the right number of supplier that a company should use is: Spend analysis. Strategic sourcing. Supply base optimization. Supply chain resilience.

Supply base optimization. Determining the right number of supplier that a company should use is supply base optimization.

The capability to resist and recover from supply chain disruptions is:

Supply chain resilience.

Which of the following describes the intersection between people, profit, and the planet?

Sustainability

What is a key factor that must be considered when an organization evaluates the potential of a project?

The business justification for investing in the project. The project's fit with overall organizational strategy. The availability of resources to execute the project.

In-line balancing:

The time at each workstation is driven by actual customer demand

What is the primary objective of line balancing

To match output rates with actual demand

The sum of all product- and logistics-related costs is:

Total landed costs.

In the United States, the majority of freight shipped is by:

Truck

In the United States, most freight is shipped by:

Truck.

Which of the following transportation modes has relatively low fixed costs but high variable costs?

Truck.

Which of the following is NOT a good reason to terminate (kill) a project?

Uncertainty regarding the project's outcomes.

The two types of data involved in quality control are:

Variables and attributes.

All of the following typically use a "make to stock" market orientation except: A. Men's jeans B. Televisions C. Coffee makers D. Wedding cake

Wedding cake

Corporate strategic planning involves decisions related to:

What businesses should we be in?

A primary reason for using global suppliers is to:

Work with the same suppliers in many different regions of the world.

How do you choose which task to assign first in a line?

You can choose to start with the task with the longest operating (takt) time or start with the task with the most number of followers

Zanda Corp. and Jones Corp. are identical in every way (products produced, costs, demand, etc.) except for one. Zanda uses a level production plan while Jones prefers a chase production plan. Which of the following is most likely to be true?

Zanda will have higher inventory carrying costs

make or buy decision

considers insourcing or outsourcing the production of parts and components.

communications technologies

create greater connectivity and speed flow of richer forms of information (e.g. the Internet)

The longest lead-time path in the BOM is the:

cumulative lead time

Service factory

facilities and equiptment represent a large portion of costs

product families

groups of products that have similar processing requirements

Which of the following is NOT a role of inventory?

increasing quality of finished goods

Spend analysis

is a process used to understand what purchases are being made, at what price, and from which specific suppliers.

Competitive bidding

is used when price is the most important factor, the specifications are known and clear, the spend level is large enough, and there are a number of equally qualified suppliers who are willing to compete.

Which is false with respect to a process structure?

its same across the entire supply chain

Which of the following tools is most appropriate when we are interested in determining if a process that was generating attribute-type data (pass/fail, good/bad) was under control?

p control chart

A process having the same activity being performed by two or more resources simultaneously is said to have what kind of structure?

parallel

back-office processes

processes that are not seen by the customer

front-office processes

processes that have contact with the customer

customer contact

the presence of the customer in a process

line balancing

used to assign tasks so that idle time and the number of workstations are minimized

communication technologies

Create greater connectivity and speed flow of richer forms of information

The focus of the first step in the DMAIC process is on:

Critical-to-quality characteristics.

The initial focus of the DMAIC process is on:

Critical-to-quality characteristics.

"Zero defects" is a concept of quality management of which thought leader?

Crosby

_____ combines break-bulk and consolidation activities.

Cross-docking

Which of the following factors was most responsible for the breakdown of the Daimler-Benz/Chrysler merger? Technology incompatibility Lack of sustainability Cultural incompatibility Different languages

Cultural incompatibility

Which of the following factors was most responsible for the breakdown of the Daimler-Benz/Chrysler merger? Technology incompatibility Lack of sustainability Cultural incompatibility Different languages

Cultural incompatibility The Chrysler/Daimler-Benz failure is an oft-cited example of a clash of cultures.

Which of the following sustainability issues has a significant impact on supply chain relationships? Governmental regulations and tariffs Location Culture Common usage of lean systems

Culture discussed in this section, culture, or more correctly similarity of culture, has a significant impact on the ease with which companies can establish and maintain supply chain relationships.

The longest lead-time path in the BOM is the:

Cumulative lead time.

The use of technology to collect and analyze data concerning customers' buying behavior is an important aspect of:

Customer Relationship Management

Jones Company makes every effort to consistently meet the expectations of all of its customers. It is clear that Jones Company is focused on:

Customer Satisfaction

The difference between a "customer success" focus and a "customer satisfaction" focus is that:

Customer satisfaction deals with expectations, customer success deals with requirements.

Which of the following challenges are more likely to be faced by service operations managers, rather than manufacturing operations manager?

Customers are more directly involved in service processes, so their perceptions need to be consideredIt is a way to categorize elements of work activities

Competitive bidding should be used when: A) There is only one qualified supplier. B) Product innovation is more important than price. C) The spend is small. D) Product specifications are known and clear

D

Qualitative factors that should be assessed when making an insourcing/outsourcing decision include all of the following EXCEPT: A) Compatibility of the supplier's organizational culture and values with your company B) The importance assigned to the supplier's location C) The skills and knowledge of the supplier's management team D) The costs incurred at the start of the contract

D

Stainless steel is an important raw material for an appliance company. Steel accounts for a high level of spend and is critical to customer satisfaction. Further, only three suppliers worldwide can produce steel to meet the company's quality standards. Which is appropriate in this situation? A) Using electronic catalogs for steel. B) Finding substitute materials. C) Using competition to select suppliers. D) Building collaborative partnerships with suppliers.

D

The process of understanding how a firm is spending its money and with which suppliers is called: A) Strategic sourcing. B) Make or buy analysis. C) Market analysis. D) Spend analysis.

D

Which activity is most likely to be outsourced by an automotive company such as Ford Motor Co.? A) The concept design for future cars. B) Final car assembly. C) Design of the engine. D) Production of tires

D

Which approach is likely to be used for leverage purchases? A) Purchasing cards B) Vendor-managed inventory C) Using executive champions D) Standardizing purchases

D

Which of the following is NOT true with respect to an RFQ or RFP? A) It describes the purchase requirements. B) It is the first step in competitive bidding. C) It is often communicated to suppliers electronically. D) It is used to determine weights in supplier scorecards.

D

Which of the following is true about the use of weighted scoring models for supplier selection? A) Once developed, the model weights should never change. B) The supply management department should determine the weights. C) The highest-scoring supplier should always be selected to receive the business. D) There is a good deal of subjectivity in developing and using the model.

D

Which of the following would NOT be a step in conducting an insourcing/outsourcing analysis? A) Assessing quantitative costs of outsourcing B) Evaluating new suppliers who could make the SKU C) Assessing the relationship of the product to the firm's core competencies D) Conducting a detailed internal audit of purchasing practices

D

Jones Company uses widgets as component parts. In a particular week it has gross requirements for 250 units of widgets, beginning inventory of 50 widgets, and scheduled receipts of 350 widgets. Available inventory for the next period is: A) 650 B) 50 C) 250 D) 150

D) 150

Advanced planning and scheduling systems: A) Use logic similar to the logic found in MRP B) Consolidate all business planning systems and data throughout an organization C) Integrate materials and capacity planning into one system D) A and C

D) A and C

After you have drawn the "current state" process map, you decide that you need a tool for showing the actual moves of material (and their locations) that take place in the manufacture of Widget A. Which of the following tools would be most appropriate? A) A "future state" map B) A process flow table C) An assembly process chart D) A physical layout diagram

D) A physical layout diagram

An example of a short-term capacity decision is: A) Adding specialized labor. B) Installing new equipment C) Adding new facilities D) Adding low-skilled labor

D) Adding low-skilled labor

Why is product quality sometimes poorly defined in a firm? A) Managers do not know how to measure customer service B) Holistic measures of quality are difficult to develop C) Managers in different functional areas tend to emphasize different dimensions of quality D) Both B and C are correct

D) Both B and C are correct

ISO 9000 defines a set of internationally accepted standards for: A) Product quality B) Quality control C) Operations management quality D) Business quality management

D) Business quality management

The longest lead time path in the BOM is the: A) Time bucket B) Planning horizon C) The window D) Cumulative lead time

D) Cumulative lead time

Variability as the source of most quality problems was a major contribution of which quality management thought leader? A) Juran B) Imai C) Crosby D) Deming

D) Deming

Software that consolidates all of the business planning systems and data throughout an organization is: A) MPS B) MRP C) DRP D) ERP

D) ERP

All of the following are potential benefits of decoupling front and back offices in service processes EXCEPT: A) Standardization across multiple locations B) Economies of scale C) Use of remotely located back office employees D) Easier customization of services

D) Easier customization of services

An example of an integrative technology is a/an: A) Transportation management system B) Industrial robot C) Radio frequency identification (RFID) system D) Enterprise resource planning (ERP) system.

D) Enterprise resource planning (ERP) system.

___________ is achieved by combining several small shipments from one shipper that are going to the same market area into one shipment.

D) Market area consolidation

Zanda Corp. wants to have a process structure that has cost advantages similar to continuous or repetitive processes, but wants to produce greater variety than those processes normally allow. Zanda should consider: A) Cellular manufacturing B) Job shop processes C) Project process D) Mass customization

D) Mass customization

John Jones, senior VP for Zanda Corp., is looking at three alternative aggregate production plans for the next six-month period. At his company, demand varies by month with substantial month-to-month differences. The three alternatives are a "pure level plan," which keeps an absolutely constant workforce, a "pure chase plan" relying on hiring and layoffs, and a hybrid plan. He is most likely to find that the pure level plan: A) Has lower inventory carrying cost than the hybrid plan B) Has higher hiring/layoff cost than the chase plan C) Has higher hiring/layoff cost than the hybrid plan D) None of these

D) None of these

Which is the most important category (as evaluated in terms of weightings used by the outside assessors) of the Malcolm Baldrige National Quality Award Program? A) Leadership B) Strategic Quality Planning C) Management of Process Quality D) None of these is the most important for the award

D) None of these is the most important for the award

Which of the following is NOT an input into the MRP? A) Bill of materials B) Inventory records file C) Master production schedule D) Planned order release

D) Planned order release

The types of costs included in a cost of quality analysis include: A) Production, delivery, and marketing B) Internal and external C) Direct and indirect D) Prevention, appraisal, and failure

D) Prevention, appraisal, and failure

Suppose that you are a process manager in a manufacturing plant, and you suspect that there is a relationship between the room temperature of the plant and the weights of the plastic caps produced by an injection molding process. What quality improvement tool would you use to investigate this claim? A) Fishbone diagram B) Check list C) Pareto analysis D) Scatter diagram

D) Scatter diagram

Which of the following is an example of an activity that is "necessary but not value-adding"? A) Schedule B) Processing C) Inspections D) Setups

D) Setups

Hugging in a control chart usually indicates: A) The process is out of control B) Systematic problems in the process that require attention C) The process is in control D) Something is occurring that is masking natural variation

D) Something is occurring that is masking natural variation

The 'triangle' symbol indicates what type of activity in a process? A) Operation B) Transportation C) Delay D) Storage

D) Storage

A manager is currently unhappy with the capabilities of a particular process and wants to improve its capabilities. To accomplish this, the manager needs to focus on the process: A) Inputs, outputs, and flows B) Activities C) Management metrics D) Structure

D) Structure

A process can be said to be capable when: A) The Cp is greater than 1.5 B) The Cpk is less than 1.0 C) The process is perfectly centered within the specification D) The Cpk is greater than 1.5

D) The Cpk is greater than 1.5

If a make-to-stock manufacturing firm with highly seasonal demand follows a chase demand strategy, which of the following is likely to be true? A) Inventory will fluctuate significantly during the year B) The production rate must be set equal to the demand in the heaviest demand period, and it must stay at that level all year. C) It will be easy to keep the workforce size stable D) The firm likely will have higher capital investment than if it followed a level plan

D) The firm likely will have higher capital investment than if it followed a level plan

You have been given the following sample information that has been drawn from the last sample taken: Mean of the sample: 12.50 Lower control limit for the range: 0 Upper control limit for the range: 10.6 Observations: 13, 12, 8, 10, 11, 16, 17, 8, 17, 18, 20, 14 Which of the following conclusions does this data support? A) The process is under control B) The process is not under control. C) The sample is within specifications D) The sample is out of specification

D) The sample is out of specification

Why are processes that are visible to the customer considered to be critical? A) They often exhibit the greatest variance (thus affecting customer waiting times). B) They are often bottlenecks (thus causing customer delays). C) They are often shared processes. D) They affect the customer perceptions of value.

D) They affect the customer perceptions of value.

You are given the following information. Which of these statements can you support with this information? Maximum Capacity: 480 hours per week Effective Capacity Ratio: 85% Demonstrated Capacity: 380 hours per week over the past two weeks On-time Delivery %: 75% of the jobs are being completed on-time A) More capacity needs to be added in the short term to improve performance in the system B) We have to look at the rate at which jobs are being input into the shop C) Our workforce is not working hard enough D) We need to further investigate whether the problem is in input or capacity

D) We need to further investigate whether the problem is in input or capacity

"Bounding" a process is necessary because: A) It determines which activity is the bottleneck B) If the project is "shelved" during your analysis, someone else may have to pick up where you left off. C) Bounding determines if a process is a core competency D) Without bounding, it is difficult to determine the beginning and ending points for the analysis.

D) Without bounding, it is difficult to determine the beginning and ending points for the analysis.

You are shipping 200 diamonds to one customer located 2,000 miles away. The value of a diamond is $1,500. You can ship via air for $500 and the diamonds will arrive in two days, or you can ship via a specialty ground carrier for $250 and the diamonds will arrive in four days. You figure your inventory carrying cost is 25 percent. Your customer will immediately transfer funds to your bank account on receipt of the shipment. Using the air carrier would save approximately how much money in total cost? Use 365 days as the number of days in a year. A. Nothing, it would actually cost more. B. $150.00 C. $247.95 D. $160.96

D. $160.96 Cost of air = $500 + ($300,000 × 2/365 × .25) = $910.96 Cost of ground = $250 + ($300,000 × 4/365 × .25) = $1,071.92.Air will save $1,071 - $910.96 = $160.96.

Which of the following types of businesses would most likely use a batch process structure?

D. A company that produces automobile seats

An example of a short term capacity decision is:

D. Adding low-skilled labor.

Process thinking causes managers to address critical process elements, including: A. Activities B. Inputs and Outputs C. Flows, structure, resources, and metrics. D. All of these.

D. All of these

Zanda Corp. made a decision to use a more expensive mode of transportation to deliver orders to customers. This decision was likely made after an examination of A. Cost-to-cost tradeoffs. B. Consolidation strategies. C. Product availability problems. D. Cost-to-service trade-offs.

D. Cost-to-service trade-offs. Increasing cost to improve service performance represents a cost-to-service trade-off.

The idea that variability is the source of most quality problems was a major contribution of which quality management thought leader? A. Juran B. Imai C. Crosby D. Deming

D. Deming

All of the following are potential benefits of decoupling front and back offices in service processes EXCEPT:

D. Easier customization of services

An example of an integrative technology is a/an:

D. Enterprise resource planning (ERP) system.

Random Corp. offers logistics services such as storage, transportation, and information processing to its customers. Random is known in industry as a(n): A. Contract warehouse. B. Common carrier. C. Public warehouse with benefits. D. Integrated service provider.

D. Integrated service provider.

For which market orientation is accurately forecasting the amount of finished goods needed likely to be the most important for a firm's financial performance?

D. Make to stock

Joe Jones, plant manager at Waco Industries, told a friend that if it was necessary, his plant could produce 1000 items a day if all conditions were just right. Joe is describing his plants: A. Effective Capacity B. Yield Rate C.Utilization D. Maximum Capacity

D. Maximum Capacity

Which of the following statements is NOT consistent with core values of TQM? A. Front line workers usually have the best ideas for solving problems. B. Every employee has a stake in product quality. C. Managers should support workers, not the other way around. D. Opinions are as important as data.

D. Opinions are as important as data.

Xanadu Inc. decided to increase the training received by new employees. The expense of this training is an example of which of the following costs of quality? A. Appraisal costs B. External failure costs C. Internal failure costs D. Prevention costs E. Quality control costs

D. Prevention costs (Training is an expense aimed at defect prevention.)

Which of the following quality improvement tools is not a particularly good brainstorming tool? A. Fishbone diagrams B. Ishikawa chart C. Cause-and-effect analysis D. Process capability analysis

D. Process capability analysis (Fishbone diagrams, Ishikawa charts, and Cause-and-effect analysis are each used as tools to stimulate discussion in brainstorming sessions.)

A manager is currently unhappy with the capabilities of a particular process and wants to improve its capabilities. To accomplish this, the manager needs to focus on the process: A. Inputs, outputs, and flows B. Activities C. Managment Metrics D. Structure

D. Structure

A process can be said to be capable when: A. The Cp is greater than 1.5. B. The Cpk is less than 1.0. C. The process is perfectly centered within the specification. D. The Cpk is greater than 1.5.

D. The Cpk is greater than 1.5.

An organizational culture is shaped by: A. The experiences of certain charismatic leaders B. Leadership memos and meeting statements C. The organization's value statement or motto D. The actions taken by leadership

D. The actions taken by leadership

A firm has an existing process that it would like to use for producing a part it will sell to an industrial customer. The customer requires that defective (out of specification) parts be kept to 3,000 parts per million (three-tenths of 1 percent) or less. A process capability study done on the process revealed that it has a process capability index (Cpk) of 1.00 for this part. What should the firm conclude from the study? A. The process is not capable, but with careful process control, it could be used to meet the customer's requirements. B. The process is highly capable, and when operating in control, a very high percentage of output will meet the customer's requirements. C. The process is not capable, and even with careful control, it will not be able to meet the customer's requirements. D. The process is capable, but just barely, and will require careful monitoring in order to meet customer requirements.

D. The process is capable, but just barely, and will require careful monitoring in order to meet customer requirements. (A Cpk score of 1 means that this process is capable, but just barely.)

With respect to transportation regulation today in the United States: A. There is no economic regulation of transportation. B. There is more economic regulation than ever before. C. There is very little safety/social regulation. D. There is more safety/social regulation than ever before.

D. There is more safety/social regulation than ever before. There is still a small amount of economic regulation, but safety and social regulation has increased dramatically.

The objective of the "control" step in the DMAIC process is: A. To consider differing opinions regarding the root causes of problems B. To create controlled experiments to analyze sources of variation C. To be rigorous in following the DMAIC steps D. To prevent an improved process from becoming highly variable again.

D. To prevent an improved process from becoming highly variable again.

You are given the following information. Which of these statements can you support with this information? Maximum Capacity: 480 hours per week Effective Capacity ratio: 85 percent Demonstrated Capacity: 380 hours per week over the last 2 weeks On-time Delivery % : 75 percent of the jobs are being completed on time

D. We need to further investigate whether the problem is in input or capacity

The positioning and replenishment of finished goods inventories at the retail level can be determined using:

DRP (distribution requirements planning)

The positioning and replenishment of finished goods inventories at the retail level can be determined using:

DRP.

Zanda Company is looking for a requirements planning system to help plan replenishment of its finished goods throughout its network of distribution centers. The type of system Zanda should consider is known as:

DRP.

An example of strategic planning is:

Deciding where to locate a new manufacturing plant.

Which of the following effects directly results from bottlenecks?

Decrease in output and increase in lead-times

Which of the following two effects is directly a result of bottlenecks?

Decrease in output and increase in lead-times.

At the heart of the Six Sigma program is a five-step process called D-M-A-I-C. These letters denote a major activity that must be completed to achieve the objectives of Six Sigma. These activities are:

Define, Measure, Analyze, Improve, Control.

Nearly all projects seem to go through the same phases in the following sequence:

Definition, planning, execution, and completion

Which process activity is due to other factors (e.g., insufficient operating capacity, lack of material, equipment breakdowns)?

Delay

Which process activity is due to other factors (e.g., insufficient operating capacity, lack of material, equipment breakdowns)? Delay Operation Storage Inspection

Delay

You walk into a hair stylist shop. All stylists are busy and you sit in the waiting area. You are in which of the following process activities?

Delay

You walk into a hair stylist shop. All stylists are busy and you sit in the waiting area. You are in which of the following process activities? Operation Delay Storage You are not in a process activity

Delay

The idea that variability is the source of most quality problems was a major contribution of which quality management thought leader?

Deming

Variability as the source of most quality problems was a major contribution of which quality management thought leader?

Deming

Which of the following forms of capacity can only be determined AFTER the completion of activities? Demonstrated capacity Maximum capacity Design capacity Effective capacity

Demonstrated capacity

What decisions do supply managers make that potentially impact quality?

Design of contracts and associated incentives and penalties

When buying from suppliers, what decisions do supply managers make that potentially impact quality?

Design of contracts and associated incentives and penalties

If a product includes features that customers care most about, we would say that it has high:

Design quality

If a product includes features that customers care most about, we would say that it has high:

Design quality.

A supply chain is a global network of organizations and activities involved in:

Designing, transforming, consuming, and disposing of goods and services.

Which of the following is NOT a component of the business model?

Desired outcome statement

The objective of statistical process control (SPC) is to:

Detect assignable cause variations versus normal random variations in the process.

Downcycling, as described in the chapter, is a critical consideration during which stage of the product life cycle? Extraction Packaging/transportation Usage Disposal

Disposal

The total landed cost A) Is the sum of all product-related costs B) Is the sum of all logistics-related costs C) Is the most important strategic decision companies should consider D) Can affect the decision of where to locate manufacturing plants E) All of the above

E) All of the above

Suppose that you are a process manager in a manufacturing plant, and you suspect that there is a relationship between the room temperature of the plant and the weights of the plastic caps produced by an injection molding process. What quality improvement tool would you use to investigate this claim? A. Fishbone diagram B. Check list C. Pareto analysis D. Histogram E. Scatter diagram

E. Scatter diagram (A scatter diagram will graphically illustrate the relationship between two variables.)

Software that consolidates all of the business planning systems and data throughout an organization is:

ERP.

Negotiation is typically used when:

Early supplier involvement is needed in new product development.

All of the following are potential benefits of decoupling front and back offices in service processes EXCEPT: A. Standardization across multiple locations. B. Economies of scale. C. Use of remotely located back office employees. D. Easier customization of services

Easier customization of services

Which of the following statements about economies of scale is NOT true?

Economies of scale refers to the fact that as volume increases, total cost of production decreases

Which of the following statements about economies of scale is NOT true?

Economies of scale refers to the fact that as volume increases, total cost of production decreases.

Which of the following statements about economies of scale is NOT true? One reason economies of scale occur is because fixed costs can be spread over more units of production as output increases One reason economies of scale occur is because employees become more efficient as volume increases Economies of scale refers to the fact that as volume increases, total cost of production decreases. Economies of scale may not exist at all levels of production

Economies of scale refers to the fact that as volume increases, total cost of production decreases.

The increasing size of container ships is being driven by:

Economies of scale.

Efficiency Equation

Efficiency = [(Sum of all task times)/(Actual work stations x takt time)] x 100

Which of the following statements that might be made by a business executive best expresses the overall philosophy of Total Quality Management?

"There's always room for improvement."

Suppose that a producer of 50 pound bags of potting soil decides that bag weight variation within a range of +/- 2 pounds will be acceptable to customers. To deliver a six sigma level of quality for this product, the bagging process will need to have a standard deviation not larger than:

0.33 pounds current = 2+2 = 4 12 deviations = 4/12 = .33

Companies that provide a bundle of logistical services to their customers are called:

3PLs.

A process with Six Sigma quality is expected to produce how many defects? A) 3.4 defects per million opportunities B) 6 defects per million opportunities C) 66,807 defects per million opportunities D) Zero defects

A) 3.4 defects per million opportunities

Which of the following process structures can produce the widest variety of products?

A. Job Shop

Compared to a functional layout, cellular layouts have:

A. Less work-in-process inventory

The use of a tablet computer such as an iPad for a restaurants wine list is an example of:

A. Mobility

To determine whether two variables are related to each other you would use: A. Scatter diagram B. Pareto analysis C. Moment of Truth analysis D. Histograms E. Capability analysis

A. Scatter diagram (Scatter diagrams help determine whether two different variables are related to each other.)

A relationship with a supplier that is characterized by high levels of distrust, little communication, and short-term transactions is called a(n): Adversarial relationship. Arms-length relationship. Acceptance of mutual goals. Full partnership.

Adversarial relationship. The characteristics describe an adversarial relationship (see Figure 10-4).

If you are evaluating whether a supplier's workforce is receiving fair wages, you are most likely doing a(n):

Assessment of sustainability.

processing technologies

Automate material and data processing to provide 24/7 resource availability, faster processing, greater consistency, lower cost

Sustainability: A) Focuses primarily on increasing environmental compliance. B) Can lower a company's total cost. C) Does not include worker safety. D) Focuses only on social issues

B

The initial focus of the DMAIC process is on: A) Process variability B) Cost C) Critical-to-quality characteristics D) Problem solving

C) Critical-to-quality characteristics

Which of the following is NOT included in the information about the inventory records file for an item? A) Safety stock B) Preferred order quantity C) Net requirements D) Lead time

C) Net requirements

The Taguchi methods link process design to: A) Customer expectations B) Production availability C) Process capability D) All of these

C) Process capability

A company has determined the ideal number of locations by looking at the trade-offs described the text. It decides to open more warehouse locations than this "ideal." This decision would most likely result in ___________ and ___________. A. Poorer service to customers and higher cost B. Better service to customers and less cost C. Better service to customers and higher cost D. Poorer service to customers and lower cost

C. Better service to customers and higher cost

Which activity at a university is the least likely to be outsourced?

Classroom instruction.

Underlying the success of companies such as Apple and Zappos is which of the following elements?

Culture

Which of the following sustainability issues has a significant impact on supply chain relationships? Culture Common usage of lean systems Location Governmental regulations and tariffs

Culture

Which of the following types of businesses would most likely use a batch process structure? A) Company that builds cruise ships B) An automotive assembler such as Ford Motor Co C) A glass beverage bottle manufacturer D) A company that produces automobile seats

D) A company that produces automobile seats

The increasing size of container ships is being driven by:

Economics of Scale

"When operational capabilities are consistent with and supportive of the value proposition and the outcomes desired by critical customers" - this statement best describes the concept of:

Fit

Customer relationship management attempts to:

Ensure the development of strategically appropriate relationships with customers.

supply management goals

Ensure timely availability of resources. Identify, assess, and mitigate supply chain risk. Reduce total costs. Enhance quality. Access technology and innovation. Foster sustainability.

ISO 14000 deals with which of the following?

Environment

Which of the following statements best describes the implementation of environmental sustainability?

Environmental sustainability always has some form of environmental impact.

When people and profit intersect, from a sustainability perspective, which of the following areas is most impacted? Equitability Sustainability Bearability Viability

Equitability

When people and profit intersect, from a sustainability perspective, which of the following areas is most impacted? Sustainability Bearability Viability Equitability

Equitability

Which of the following process structures can produce the widest variety of products

Job Shop

Which of the following is NOT an input into the S&OP?

Lot Sizing Logic.

Which of the following is an advantage of a chase production strategy (as compared to a level plan)?

Lower inventory investment.

The Williams Co. combines all of its orders going to Tokyo, Japan, during the month into a single shipment. This is an example of:

Market area consolidation.

Make/buy/subcontract is typically considered at what time horizon when doing capacity planning?

Medium-term

Which of the following is NOT included in the information about the inventory records file for an item?

Net requirements

Zanda Corp. had outsourced its production to a company located in Asia. Recently it decided to continue to outsource but bring the production back to a company located in the United States. This decision was likely made after Zanda:

Examined total cost of ownership.

Zanda Corp. had established a production plant in Thailand. Recently it decided to close the plant and move production to Mexico. This decision was likely made after Zanda:

Examined total landed cost.

A common goal when designing a functional layout is to

Reduce the time and cost of moving people and materials between departments

A common goal when designing a functional layout is to

Reduce the time and cost of moving people and materials between departments.

The main benefit of integrated/concurrent product development is:

Reduced launch and production costs.

If a tangible good or service performs the promised function dependably, we say that it has high:

Reliability.

Companies with a make-to-stock orientation are most likely to use which of the following process structures?

Repetitive Process

Which of the following are factors affecting the success of a project?

Experience and authority of team members. Organizational structure and reward systems. Equipment, facilities, and communications systems.

Demand-driven MRP:

Extends planning across the supply chain.

Quality dimensions are easy to measure for tangible goods, while they are difficult to measure for services. TRUE OR FALSE

FALSE

The focus fo an aggregate production plan, in general, is on all the following EXCEPT:

Facilities and capital equipment

The focus of an aggregate production plan, in general, is on all of the following EXCEPT:

Facilities and capital equipment.

If a company spends more on prevention, what would be the expected impact on other costs of quality?

Failure costs will go down.

Which of the following programs/initiatives strives to improve the lives of suppliers and their families? ISO 14000 Responsible Care LEED FairTrade

FairTrade

_______ refers to establishing specific times when deliveries will be made to customers.

Scheduled delivery consolidation

Which of the following programs/initiatives strives to improve the lives of suppliers and their families? Responsible Care ISO 14000 LEED FairTrade

FairTrade Of the various initiatives, only Fair Trade deals with suppliers and their conditions

The carbon footprint is established by estimating the difference between direct and indirect greenhouse gas emissions?

False

Bill wants to purchase a new machine that is expected to hold tighter tolerances on production parts. What functional managers are likely to have a stake in this decision?

Finance, Manufacturing, Engineering

____________ inventory is the costliest form of inventory one can hold

Finished goods

Make to stock (MTS)

Finished goods that are held in inventory in advance of customer orders - firms make forecast of customer demand - standardized, mature products - repetitive assembly lines, continuous processes (books, electronics, cars)

From an operations standpoint, sustainability means that: Profit is not an important goal. A company should focus its efforts on reducing pollution. Firms should pursue profit without damaging the environment or the well-being of future generations. The firm must improve how it manages its people resources since these are important to the long-term survival of the firm.

Firms should pursue profit without damaging the environment or the well-being of future generations.

Operations managers answer questions of what, how, when, where, and who by defining both the ___________ and _____________ aspects of the operations management system.

Structural and infrastructural

A company has decided that it no longer needs to extensively count and inspect the products it receives from a particular supplier. This suggests that the purchasing company has begun: Spend analysis. Supplier certification. Process simplification. Time reduction analysis.

Supplier certification. After certifying a supplier, a company may decide it no longer needs to perform certain activities such as counting and inspecting.

The identification, acquisition, positioning, and management of resources and capabilities that a firm needs to attain its strategic objectives is:

Supply management.

Constant changing of the master schedule and the resulting changes in the requirements for components is referred to as:

System nervousness.

From an operations standpoint, sustainability means that: Firms should pursue profit without damaging the environment or the well-being of future generations. Profit is not an important goal. The firm must improve how it manages its people resources since these are important to the long-term survival of the firm. A company should focus its efforts on reducing pollution.

Firms should pursue profit without damaging the environment or the well-being of future generations. Sustainability is much more than reducing pollution, and it includes meeting profitability goals. Firms should pursue profit without damaging the environment or the well-being of future generations.

Zanda Corp. has determined that it has too many products returned from customers. Zanda is interested in determining what factors or elements could contribute to the large number of returns. Which of the following quality tools would Zanda use to achieve this objective?

Fishbone diagram

Outsourcing is least likely when:

The process in highly innovative.

There are _____ basic transportation modes or methods of transporting items.

Five

You have been given the following sample information that has been drawn from the last sample taken: Mean of the sample: 12.50 Lower control limit for the range: 0 Upper control limit for the range: 10.6 Observations: 13, 12, 8, 10, 11, 16, 17, 8, 17, 18, 20, 14 Which of the following conclusions does this data support?

The sample is out of specification.

When a process is automated:

Fixed costs increase

When a process is automated:

Fixed costs increase.

Ensuring that all the right people, equipment, and materials arrive on time is especially challenging when using which layou

Fixed-Position Layout

Ensuring that all the right people, equipment, and materials arrive on time is especially challenging when using which layout?

Fixed-Postion Layout

. Ensuring that all the right people, equipment, and materials arrive on time is especially challenging when using which layout?

Fixed-position Layout

Which of the following is considered a "process-related" competitive priority?

Flexibility

Systems that combine automated machines, robots, and material handling systems and are controlled by a single computer are

Flexible manufacturing systems (FMS)

Systems that combine automated machines, robots, and material handling systems and are controlled by a single computer are:

Flexible manufacturing systems (FMS).

The total time that it takes to get one unit through a process is called:

Flow time

leverage

For ___ purchases, annual contracts are often used.

strategic

For ___ purchases, longer-term contracts lasting through the product's life are the norm

supply chain resilience

For high likelihood/high impact risks, firms invest to increase ___ ___ ___

All of the following are outputs from an MRP system EXCEPT:

Forecasts.

Decision support system, processing technologies, communication technologies, integrative technologies

Four types of supply chain operational technologies

Core values of TQM?

Front line workers usually have the best ideas for solving problems. Every employee has a stake in product quality. Managers should support workers, not the other way around. Data provide a better basis for decision making than opinions.

A College of Business building that has classrooms on the first floor, faculty offices on the second floor, and graduate assistance offices on the third floor has which type of layout?

Functional

Sustainability metrics are being added to operations management both as customer requirements and as operational performance criteria.

True

Bottleneck purchases

are high risk and low spend and typically are not core to the firm's performance, but lack of availability can cause delays.

Xanadu Inc. decided to increase the training received by new employees. The expense of this training is an example of which of the following costs of quality: Appraisal costs External failure cost Prevention costs Quality control costs

c

interactive technologies

combine data management, communications, decision support, and processing capabilities (e.g. Cloud computing)

ISO 9000 defines a set of internationally accepted standards for: Product quality Quality control Operations management quality Business quality management

d

Maxmo Corporation identified the following costs for the most recent fiscal year: Vendor evaluation = $20,000 Design engineering = $100,000 Scrap = $15,000 Training = $30,000 Warranty = $40,000 What was Maxmo's total prevention cost for the year? $205,000 $85,000 $55,000 $150,000

d

make to stock (MTS)

finished goods that are held in inventory in advance of customer orders (e.g. groceries, retail clothing, and electronics)

professional service

high labor intensity, high customization/customer interaction (lawyers, doctors, accountants, and consultants)

customer action

include all the steps that customers take as part of the service delivery process (call room service)

Price

is the focus in adversarial or arm's-length relationships.

Supply management

is the identification, acquisition, positioning, and management of resources and capabilities that a firm needs to attain its strategic objectives.

Joe Jones, plant manager at Waco Industries, told a friend that if it was necessary, his plant could produce 1,000 items a day if all conditions were just right. Joe is describing his plant's:

maximum capacity

part 2

part 2

precedence relationships

presents the order in which tasks must be completed

physical evidence

represents all the tangibles that customers see or collect during their contact with a company (menu)

Texas Manufacturing has decided to triple the number of distribution centers it operates around North America in order to locate inventories within 500 miles of each major city. What effects should they expect?

required safety stock will increase

front-office contact employee actions

the actions of frontline contact employees that occur as part of a face-to-face encounter with customers (greet and take bags)

Noncritical items

typically are a low percentage of overall spend and have little impact on performance.

supply chain resilience

which is the capability to resist and recover from supply chain disruptions.

Bill's glass store needs to ship an order of 10 chandeliers to a builder about 1,000 miles away. The chandeliers cost about $10,000 each, and Bill will be paid upon delivery. Bill plans to ship the order by truck at a cost of $1,000. The delivery will take 10 days. Bill uses a 30 percent annual inventory carrying cost. What will be the approximate total shipping and transit inventory carrying cost of the shipment? A. $633.33 B. $821.92 C. $1,000 D. $1,821.92

$1,000 + ($100,000 × 10/365 × .3) = $1,821.92

Maxmo Corporation identified the following costs for the most recent fiscal year: Vendor evaluation = $20,000 Design engineering = $100,000 Scrap = $15,000 Training = $30,000 Warranty = $40,000 What was Maxmo's total prevention cost for the year?

$150,000

Alpha Company places 10 orders per year with its supplier. Each order is for an amount exactly equal to the EOQ. Alpha's order cost has been determined to be $50 per order. Alpha carries no safety stock at all. What is Alpha's annual inventory carrying cost? A. Cannot be determined without further information B. $50 C. $250 D. $500

$200

Johnson Company had beginning inventory of $1,000,000 and ending inventory of $1,200,000. Johnson has determined inventory carrying cost to be 25 percent. Johnson's inventory carrying cost was:

$275,000

Johnson Company had beginning inventory of $1,000,000 and ending inventory of $1,200,000. Johnson has determined inventory carrying cost to be 25%. Johnson's inventory carrying cost was:

$275,000

HighLife Corporation has the following information: Average demand = 30 units per day Average lead time = 40 days Item unit cost = $45 for orders of less than 400 units Item unit cost = $40 for orders of 400 units or more Ordering cost = $50 Inventory carrying cost = 15 percent The business year is 300 days. Standard deviation of demand during lead time = 90 Desired service level = 95 percent What is the TAC at $40/unit? (Pick the number closest to the correct answer.)

$363,325 EOQ=2(300)(30)($50)$40(.15)‾‾‾‾‾‾‾‾‾‾‾‾‾√=387.30EOQ=2(300)(30)($50)$40(.15)=387.30 However, it is necessary to order at least 400 units to obtain the $40 price. Therefore the TAC of ordering 400 units is: Inventory carrying cost = 400/2($40)(.15) = $1,200 Ordering cost = (300)(30)/400 × $50 = $1,125 Product cost = 9,000($40) = $360,000 TAC = $1,200 + $1,125 + $360,000 = $362,325

Zanda Corporation is preparing an aggregate production plan for its product for the next four months. The company's expected monthly demand is given in the following chart. The company will have 100 units in inventory at the beginning of the month and wishes to maintain at least 100 units at the end of each month. Following is other critical data: Production cost per unit = $125 Inventory carrying cost per month per unit = $10 (based on ending month inventory) Hiring cost per worker = $50 Firing cost per worker = $100 Beginning number of workers = 25 What is the total cost of a CHASE plan (using hiring / firing)?

$380,300 With a chase plan, production will be 3,000 units at $125 per unit = $375,000. Using a chase strategy you will lay off one worker in month 1 and four workers in month 3. 5 layoffs at $100 each = $500 You will hire 8 workers in month 2 and month 4. 16 workers hired at $50 each = $800 Inventory carry cost will be 100 units per month at $10 per month for 4 months = $4,000 $375,000 + $500 + 800 + 4,000 = $380,300.

Jones Corporation is preparing an aggregate production plan for washers for the next four quarters. The company's expected quarterly demand is given in the following chart. The company will have 1,000 washers in inventory at the beginning of the year and wishes to maintain at least that number at the end of each quarter. Following is other critical data: Production cost per unit = $250 Inventory carrying cost per quarter per unit = $10 (based on quarter-ending inventory) Hiring cost per worker = $1,000 Firing cost per worker = $2,000 Beginning number of workers = 10 Each worker can produce 100 units per quarter. Given this data, what is the total cost of a LEVEL plan?

$5,110,000 A level production plan is 20,000 units ÷ 4 quarters = 5,000 units per quarter. 40 workers need to be hired at $1,000 per worker = $40,000. 20,000 washers will be produced at $250 each = $5,000,000. With the 1,000 units of beginning inventory, this results in ending inventory of 2,000 | units at the end of quarter 1 and 3,000 units of inventory at the end of quarter 3. The ending inventory at the end of quarter 2 and 4 is 1,000 in accordance with the company policy of holding at least 1,000 units at the end of each quarter. The total quarterly ending inventory is, therefore 2,000 + 1,000 + 3,000 + 1,000 units = 7,000 units. At $10 per unit carrying cost per quarter, that is $70,000.Total production cost = $40,000 + $5,000,000 + $70,000 = $5,110,000.

Natalie's Cabinets makes cabinets at an average cost of $2,000. Last year, Natalie sold 5,000 units of the cabinets and had an annual turnover rate of 4 times. Natalie has estimated her inventory carrying cost to be 25 percent. What was Natalie's annual inventory carrying cost?

$625,000

Natalie's Cabinets makes cabinets at an average cost of $2,000. Last year, Natalie sold 5,000 units of the cabinets and had an annual turnover rate of four times. Natalie has estimated her inventory carrying cost to be 25 percent. What was Natalie's annual inventory carrying cost?

$625,000

What is the takt time in minutes, if 500 units are needed per day? The company works 2, 8-hour shifts per day.

(2 x 8 x 60)/(500) = 1.9

Cellular layout

- arranges workstations to form a number of small assembly lines call work cells - product families: groups of products that have similar processing requirement - flexibility of a small, focused job shop with efficiency of a repetitive line

Theoretical number of station (N)

- determine the minimum possible number of stations (Total of all task times/takt time)

Service shop

- high capital intensity, high customer interaction/customization (automobile repair shop, hospital)

Professional service

- interact closely with clients to deliver customized services - time consuming, costly (providers high skilled, educated) (Lawyer, doctors, consultants)

Service factory

- low customer contact, customization, labor intensity while investment in facilities and equipment is high (airline, hotels, trucking companies)

Mass service

- low customer interaction/customization, high labor intensity (retail banks, gas station)

Efficiency

- to improve efficiency, reduce time at the bottleneck station ([sum of all task times/(actual workstations x takt time)] x 100)

Forty samples of 100 are taken, with the total number of defective units being 150. What is the lower control limit for the three sigma (z = 3) p-chart?

0.000

Forty samples of 100 are taken, with the total number of defective units being 150. What is the upper control limit of the three sigma (z = 3) p-chart?

0.0945

The managers of a production line that fills cereal boxes would like to reach a Six Sigma level of quality in terms of product weight. The desired range of output extends from 10 to 16 ounces. In order to reach Six Sigma quality, what would the maximum standard deviation of output need to be?

0.5 ounces

The managers of a production line that fills cereal boxes would like to reach a Six Sigma level of quality in terms of product weight. The desired range of output extends from 10 to 16 ounces. In order to reach Six Sigma quality, what would the maximum standard deviation of output need to be?

0.5 ounces 10-16 = 6 deviations of 1 ounce Six sigma is 12, so 12 deviations of .5 ounces

The specifications for the diameter of a molded part are 10 mm ± 0.5 mm. The actual average and standard deviation from 250 parts sampled is 10.1 mm and 0.1 mm, respectively. What is the process capability as measured by Cp?

1.67

XYZ distribution center receives 1,000 deliveries each day. Arrivals are on time if they are within plus or minus 10 minutes of the targeted arrival. In order to have only 3.4 late or early arrivals per million deliveries, what would the standard deviation of arrival times need to be?

1.67 minutes

XYZ distribution center receives 1,000 deliveries each day. Arrivals are on time if they are within plus or minus 10 minutes of the targeted arrival. In order to have only 3.4 late or early arrivals per million deliveries, what would the standard deviation of arrival times need to be?

1.67 minutes (10+10)/12 Six Sigma has 12 deviations

Sample means and ranges were obtained for five samples of 10 units per sample from a production process. Assume the process was considered to be in control during the period these samples were collected. The results are as follows: Sample # Mean Range 1 10.58 0.23 2 9.97 0.25 3 9.99 0.21 4 10.23 0.28 5 10.23 0.28 LCL for the three sigma X chart for the process is approximately (for a sample size of 10, A2 = 0.31):

10.12.

Sample means and ranges were obtained for five samples of 10 units per sample from a production process. Assume the process was considered to be in control during the period these samples were collected. The results are as follows: Sample # Mean Range 1 10.58 0.23 2 9.97 0.25 3 9.99 0.21 4 10.23 0.28 5 10.23 0.28 What is the central line of the X chart for this process

10.20

n the preceding question (below), suppose the utilization is 70 percent of effective capacity. What is the actual output of the process? The following sequence shows four operations for a computer chip assembly process and the effective capacity of each. Which step is the bottleneck? Step 1: 500 chips/hour Step 2: 250 chips/hour Step 3: 200 chips/hour Step 4: 550 chips/hour 140 chips per hour 350 chips per hour 175 chips per hour 200 chips per hour

140 chips per hour

A company must make 400 units per day. The company works one 12-hour shift. What is the efficiency if the total time to complete all tasks is 23 minutes and the company is using 18 workstations?

22%

A company needs to produce 1,000 units per day (the day is defined as 8 hours of production). There are six tasks to be completed with a total task time of 12 minutes. The company should have

25 Work Stations

A company needs to produce 1,000 units per day (the day is defined as 8 hours of production). There are six tasks to be completed, with a total task time of 12 minutes. The company should have

25 Workstations (1,000 units × 12 minutes)/(8 hours × 60 minutes) = 25

A company has average demand of 30 units per day. Lead time from the supplier averages seven days. Assume that the combined standard deviation of demand during lead time has been calculated and is equal to 20 units. One unit costs $10 and the inventory carrying cost is 25%. 1 standard deviation covers 84.13% 1.04 standard deviations covers 85% 1.28 standard deviations covers 90% 1.65 standard deviations covers 95% 1.96 standard deviations covers 97.5% 2 standard deviations covers 97.72% 2.33 standard deviations covers 99% 3 standard deviations covers 99.86% 6 standard deviations covers 99.99966% What is the reorder point for the company if it decides on a 99% service level?

257 units (30*7)+(2.33*20)=257

A process with Six Sigma quality is expected to produce how many defects?

3.4 defects per million opportunities

What is the takt time in minutes, if 500 units are needed per day? The company works 2, 8-hour shifts per day.

3.6 minutes

A company needs to produce 400 units per day (the day is defined as 12 hours of production). There are three tasks to be completed, with a total task time for all three tasks of 6 minutes. The company should have:

4 Workstations

A company needs to produce 400 units per day (the day is defined as 12 hours of production). There are three tasks to be completed, with a total task time for all three tasks of 6 minutes. The company should have:

4 Workstations (400 items × 6 minutes)/(12 hours × 60 minutes) = 3.33 (round up to 4)

The Smith Manufacturing Co. uses a weighted-point model to evaluate new suppliers. Based on its competitive priorities, quality performance is weighted 60%, Delivery performance is 25%, and cost is 15%. On a 5-point scale, a supplier is rated, 4 on quality, 5 on delivery, and 5 on cost, what is its overall weighted score? 14. 4.4. 4. 5.

4.4. The ratings are weighted by the weights and then added together. 0.6x4 + 0.25x 5 +0.15*5 = 4.4.

Use the information contained in the chart below to determine your answer. Twenty samples of size 8 were taken from a stable process. From past studies of the process, you know that the overall mean is 42.5 and that the average of the samples range is 2.0. What is the upper control limit for the x-bar chart? Picture

43.25

The following environmentally responsible process/material matrix was developed for Zanda Corp. Zanda has determined that a score of 40 is its goal at this time, but efforts to lower its score will continue regardless. What is Zanda's current score, and what is your conclusion? 53, Zanda is doing better than its goal. 53, Zanda is doing worse than its goal. 47, Zanda, is doing better than its goal. 47, Zanda is doing worse than its goal.

47, Zanda is doing worse than its goal.

The following environmentally responsible process/material matrix was developed for Zanda Corp. Zanda has determined that a score of 40 is its goal at this time, but efforts to lower its score will continue regardless. What is Zanda's current score, and what is your conclusion? 53, Zanda is doing better than its goal. 47, Zanda, is doing better than its goal. 47, Zanda is doing worse than its goal. 53, Zanda is doing worse than its goal.

47, Zanda is doing worse than its goal. The sum of the scores is 47. Since this is a higher number than the goal, Zanda is doing worse (the ultimate objective would be a score of zero).

The following environmentally responsible process/material matrix was developed for Zanda Corp. Zanda has determined that a score of 50 is its goal at this time, but efforts to lower its score will continue regardless. What is Zanda's current score, and what is your conclusion? 47, but Zanda must do something about solid residue during extraction now. 47, and Zanda has met its goals—no further actions are necessary. 15, (from manufacture) and Zanda has met its goals—no further actions are necessary. 53, and Zanda has not met its goals.

47, but Zanda must do something about solid residue during extraction now. While the sum of 47 is less than the goal of 50, the problem facing Zanda is that of solid residue during extraction. This has a score of 4, which indicates an event that is extremely dangerous and highly hazardous from an environmental perspective.

Jones Corporation is preparing an aggregate production plan for washers for the next four quarters. The company's expected quarterly demand is given in the following chart. The company will have 1,000 washers in inventory at the beginning of the year and wishes to maintain at least that number at the end of each quarter. Following is other critical data: Production cost per unit = $250 Inventory carrying cost per quarter per unit = $10 (based on quarter-ending inventory) Hiring cost per worker = $1,000 Firing cost per worker = $2,000 Beginning number of workers = 10 Each worker can produce 100 units per quarter. If Jones prefers a level plan, what will be the regular production rate per quarter?

5,000 units 20,000 units annually divided by 4 quarters is 5,000 units per quarter

A retail store's average sales are $100,000 per week. On average it holds $500,000 worth of inventory valued at cost to the retailer. The retailers' average pricing included a mark-up of 30%. The retailer's annual inventory turnover is:

8 Turnover= (52*$100,000) / ($500,000 * 1.3) = 8

A company must make 30 units per hour. What is the efficiency if the total time to complete all tasks is 8 minutes and the company is using 5 workstations?

80%

Under Juran's Law, whenever a problem occurs, what percentage of the time is the problem the result of a system/process error? 15 percent 50 percent 85 percent 100 percent

85 percent

Under Juran's Law, whenever a problem occurs, what percentage of the time is the problem the result of a system/process error?

85%

Suppose demand is 45 units a month, average inventory is 60 units, and unit cost is $20. What is the annual inventory turnover?

9

A company must make 30 units per hour. What is the efficiency if the total time to complete all tasks is 8 minutes and the company is using 5 workstations?

94%

A global food products company makes soups that are specifically tailored to the tastes of consumers in individual countries. What sourcing strategy is the company likely to use for its fresh ingredients such as vegetables? A) Local, close to its production plant when possible. B) National, using suppliers within each country. C) Global, using the same suppliers across the world. D) Low-cost country sourcing.

A

A relationship with a supplier that is characterized by high levels of distrust, little communication, and short-term transactions is called a(n): A) Adversarial relationship. B) Arm's-length relationship. C) Acceptance of mutual goals. D) Full partnership.

A

All of the following are true about supplier scorecards EXCEPT: A) They reduce the need for incoming quality inspection. B) They are used to provide performance feedback to suppliers. C) They are often used to categorize suppliers based on an overall score. D) They are used to measure supplier performance based on key performance indicators

A

In its own country, a former supplier to your company recently began selling its own product that is identical to a product that was developed by your company. This is an example of: A) Supply chain risk. B) Sustainability. C) Insourcing. D) Strategic sourcing.

A

Jones Company has identified an item for which the supply risk is high and the value of the purchase to the firm is low. A recommended sourcing strategy for Jones is to: A) Use multiple sourcing. B) Increase efficiencies. C) Build partnerships. D) Consolidate purchases

A

Local sourcing: A) Reduces delivery costs. B) Is called "nearshoring." C) Is used when cost is the primary objective. D) Takes advantage of lower trade barriers from trade agreements

A

Outsourcing is least likely when: A) The process is highly innovative. B) Products are in the mature phase of the life cycle. C) The production processes are standard. D) The activity is not strategically important

A

The identification, evaluation, selection, and management of suppliers is: A) Sourcing. B) Supply management. C) Supply chain management. D) Spend analysis.

A

Which of the following is (are) advantage(s) of full partnerships with suppliers, as compared to traditional adversarial relationships? I. They help reduce uncertainties for both buyer and supplier. II. The full partnership is easier to establish and maintain than other relationships, so more suppliers can be dealt with. III. The full partnership assures the lowest material price. A) I only B) II only C) III only D) I and II only E) II and III only

A

The Williams Manufacturing Co. uses a weighted-point model to evaluate new suppliers. Based on its competitive priorities, quality performance is weighted 40 percent, delivery performance is 30 percent, and cost is 30 percent. On a five-point scale, a supplier is rated 5 on quality, 3 on delivery, and 4 on cost. What is its overall weighted score? A) 4.1. B) 12. C) 1.2. D) 4.

A The ratings are weighted by the weights and then added together. 0.4 × 5 + 0.3 × 3 + 0.3 × 4 = 4.1.

sourcing strategy

A ___ ___ must consider: Number of suppliers to use. Capabilities and location of suppliers. Type of relationship and contract length.

Which of the following is true concerning sales and operations planning?

A benefit of the process is that the firm should achieve high service levels with lower inventory.

Which is product is likely to use a fixed positon layout in its production process? A. A refrigerator B. Customized equipment C. A commercial airplane D. A mobile phone?

A commercial airplane

Which of the following types of businesses would most likely use a batch process structure

A company that produces automobile seats

Which of the following types of businesses would most likely use a batch process structure? A. Company that builds cruise ships. B. An automotive assembler such as Ford Motor Co. C. A glass beverage bottle manufacturer. D. A company that produces automobile seats

A company that produces automobile seats

Which of the following is the best example of a service shop?

A computer repair shop

With respect to lead-time performance, most customers would prefer which of the following from a supplier?

A consistent three-day lead time

Indifference analysis

A cost-based quantitative decision-making tool that identifies the production volume at which the total costs of two processes are equal

Job shop

A flexible process structure for products that require different inputs and have different flows through the process - Customized, low volume (Auto repair, beauty salon, copy shop)

A primary advantage of a functional project organizational structure is:

A functional manager controls both the budget and the schedule.

Functional layout

A layout that groups together similar resources - job shop, batch (fitness, copy shop)

Product layout

A layout where resources are arranged according to a regularly occurring sequence of activities - repetitive, continuous processes (automobile assembly line)

The Plan-Do-Check-Act cycle is best defined as

A method used to guide problem identification and solution.

The Plan-Do-Check-Act cycle is best defined as:

A method used to guide problem identification and solution.

Project

A one-time or infrequently occurring set of activities that create outputs within pre specified time and cost schedules - Unique, one of a kind (custom home, designing a video game)

Alpha Company has a performance standard of 97 percent fill rate. Last month it achieved a 94 percent fill rate. This is an example of:

A performance gap.

You realize that you need a tool for recording all the move distances, process flow symbols, people involved, value add designations, and time required for each step to get an overall grasp for where waste exists in the process. The best tool for this would be: A value stream map A process flow table An assembly process chart A physical layout diagram

A process flow table

Repetitive process

A process in which discrete(分离的;有区别的) products flow through the same sequence of activities - Standard products with a range of options (Appliances, automobiles, buffet restaurant)

Outsourcing is often a good choice when:

A product is in the mature phase of the life cycle.

Outsourcing is often a good choice when: There are only a few suppliers. A product is in the mature phase of the life cycle. Technology is new. The product is critical to a firm's competitiveness.

A product is in the mature phase of the life cycle. Outsourcing is often a good choice for mature products. There should be many capable suppliers and known technology Critical competitive items generally shouldn't be outsourced.

inputs

A product's quality depends in large part upon the quality of all of its ____.

Continuous process

A single-flow process used for high-volume non discrete, standardized product (Job rotation) - Commodities日用品;必需品 with high volume, little variety (Aluminium cans, detergent, gasoline)

Zanda Corp. is considering trying to develop a full partnership with a particular supplier. This suggests that the item Zanda buys from the supplier is:

A strategically important part

Zanda Corp. is considering trying to develop a full partnership with a particular supplier. This suggests that the item Zanda buys from the supplier is:

A strategically important part.

A process is:

A system of activities that transforms inputs into valuable outputs.

Request for Proposal (RFP)

A type of procurement document used to request proposals from prospective sellers of products or services. In some application areas, it may have a narrower or more specific meaning.

In the preceding question (below), suppose the utilization is 70 percent of effective capacity. What is the actual output of the process? The following sequence shows four operations for a computer chip assembly process and the effective capacity of each. Which step is the bottleneck? Step 1: 500 chips/hour Step 2: 250 chips/hour Step 3: 200 chips/hour Step 4: 550 chips/hour A) 140 chips per hour B) 350 chips per hour C) 175 chips per hour D) 200 chips per hour

A) 140 chips per hour

A company needs to produce 1,000 units per day (the day is defined as 8 hours of production). There are six tasks to be completed, with a total task time of 12 minutes. The company should have: A) 25 workstations B) 2 workstations C) 84 workstations D) 14 workstations

A) 25 workstations

Which of the following products is most likely to use an assemble-to-order market orientation? A) An upholstered sofa B) A mobile phone C) Shampoo D) A race car

A) An upholstered sofa

When should the metrics used in evaluating a process be developed? A) As soon as we have identified the value proposition B) After we have identified the critical process C) When we have done the "as is" analysis of the process D) After we have completed the future state analysis of the process

A) As soon as we have identified the value proposition

"Quality at the source" suggests quality should be: A) Built into the product B) Inspected at multiple points C) Ensured by the quality control lab D) An inspector's function

A) Built into the product

Wanda Corp. currently uses a job shop process. It wants to maintain the advantages of this but increase efficiency. Wanda should investigate: A) Cellular manufacturing B) Job shop processes C) Mass customization D) Project process

A) Cellular manufacturing

Jones Company has a make-to-order orientation. It most likely does NOT use: A) Continuous process structure B) Continuous process structure C) Job shop process structure D) Jones is likely to use any of these

A) Continuous process structure

Suppose you learn that a process has a very long waiting time. Which of the following would decrease the waiting time? A) Decrease utilization B) Increase the coefficient of variation of job arrival times. C) Increase the coefficient of variation of job processing times D) None of these would decrease waiting time

A) Decrease utilization

At the heart of the Six Sigma program is a five-step process called D-M-A-I-C. These letters denote a major activity that must be completed to achieve the objectives of Six Sigma. These activities are: A) Define, Measure, Analyze, Improve, Control B) Define, Measure, Analyze, Inquire, and Compare C) Define, Measure, Analyze, Involve, and Cost D) Define, Measure Analyze, Improve, and Compare

A) Define, Measure, Analyze, Improve, Control

Which of the following forms of capacity can only be determined AFTER the completion of activities? A) Demonstrated capacity B) Maximum capacity C) Design capacity D) Effective capacity

A) Demonstrated capacity

An application of the DMAIC process to product design is known as: A) Design for Six Sigma B) Design for Quality C) Quality Function Deployment D) None of these answers are correct.

A) Design for Six Sigma

John Jones, senior VP for Zanda Corp., is looking at three alternative aggregate production plans for the next six-month period. At his company, demand varies by month with substantial month-to-month differences. The three alternatives are a "pure level plan," which keeps an absolutely constant workforce, a "pure chase plan" relying on hiring and layoffs, and a hybrid plan. He is most likely to find that the hybrid plan: A) Has lower inventory carrying cost than the level plan and lower hiring/layoff cost than the chase plan. B) Has higher hiring/layoff cost than the chase plan and higher inventory carrying cost than the level plan. C) Has lower inventory carrying cost than the chase plan and higher hiring/layoff cost than the level plan. D) None of these

A) Has lower inventory carrying cost than the level plan and lower hiring/layoff cost than the chase plan.

Most processes involve two basic types of flows: A) Information flows and physical flows B) Product flows and decision flows C) Information flows and operator flows D) Physical flows and documentation flows

A) Information flows and physical flows

If a make-to-stock manufacturing firm with highly seasonal demand follows a level production strategy, which of the following is likely to be true? A) Inventory will fluctuate significantly during the year B) The production rate must be set equal to the demand in the heaviest demand period, and stay at that level all year. C) It will be difficult to keep the workforce size stable D) The firm must make sure that its maximum capacity is at least as high as the heaviest demand period

A) Inventory will fluctuate significantly during the year

Which of the following process structures can produce the widest variety of products? A) Job shop B) Repetitive process C) Continuous flow D) Batch operation

A) Job shop

Compared to a functional layout, cellular layouts have: A) Less work-in-process inventory B) More complexity in scheduling C) More frequent setups D) Lower efficiency

A) Less work-in-process inventory

The use of a tablet computer such as an iPad for a restaurant's wine list is an example of: A) Mobility B) Point-of-sale system C) Warehouse management system D) Enterprise resource management system

A) Mobility

Production planning and scheduling is most likely to be associated with which of the following activities? A) Operations B) Inspections C) Storages D) Transportations

A) Operations

Value in a process flow analysis is most often associated with which of the following activities? A) Operations B) Inspections C) Storages D) Transporations

A) Operations

The width of a histogram showing a distribution of data indicates the __________ of the process that generated the distribution. A) Predictability B) Average C) Bias D) Central tendancy

A) Predictability

A tool used to quickly communicate proposed process changes effectively is called a: A) Process summary table B) Future state map C) Work process flowchart D) Process flow diagram

A) Process summary table

Which of the following statements is false? A) Quality dimensions are easy to measure for tangible goods, while they are difficult to measure for services B) Dimensions of service quality are similar to the dimensions of quality for tangible goods C) Some firms still have a poor understanding of quality and quality management D) All of these are false

A) Quality dimensions are easy to measure for tangible goods, while they are difficult to measure for services

Generally speaking, the sales function and operations function differ in objectives. Which of the following is NOT one of those differences? A) Sales prefers detailed forecasts for setting bonuses, operations prefers aggregate forecasts B) Sales prefers many product variations, operations prefers few variations C) Sales emphasizes revenue, operations emphasizes cost minimization D) All of these

A) Sales prefers detailed forecasts for setting bonuses, operations prefers aggregate forecasts

All of the following represent external failure cost EXCEPT the cost of: A) Scrapping defective raw material B) Repairing items covered by warranty C) Replacing defective items found by customers D) Loss of goodwill and reputation

A) Scrapping defective raw material

Choose the service type that is correctly matched with one of its key characteristics A) Service factory-facilities and equipment represent a large proportion of total costs B) Service shop-there is a low degree of customer interaction C) Mass service-transactions are very varied from customer to customer D) Professional service-transactions tend to be fairly standardized

A) Service factory-facilities and equipment represent a large proportion of total costs

If you were developing a service blueprint, how would you classify "delivery of gasoline to a service station"? A) Support processes B) Physical evidence C) Back-office/invisible contact employee actions D) Front-office/visible contact employee actions

A) Support processes

Which of the following tools is most appropriate when we are interested in determining if a process that was generating attribute-type data (pass/fail, good/bad) was under control? A) p control chart B) Xbar-R control chart C) Scatterplot D) Histogram

A) p control chart

Sample means and ranges were obtained for five samples of 10 units per sample from a production process. Assume the process was considered to be in control during the period these samples were collected. The results are as follows: Sample 1, 2, 3, 4, 5 Mean 10.58, 9.97, 9.99, 10.23, 10.23 Range0.23, 0.25, 0.214, 0.28, 0.28 LCL for the three sigma X chart for the process is approximately (for a sample size of 10, A2 = 0.31): Multiple Choice A. 10.12. B. 10.05. C. 10.94. D. 9.44. E. 9.75.

A. 10.12. (Average range per sample = 0.25 × 0.31 = 0.0775 and 10.2 − 0.0775 = 10.1225)

In the preceding question, suppose the utilization is 70 percent of effective capacity. What is the actual output of the process?

A. 140 chips/hour 200(.7)=140

A company needs to produce 1000 units per day(the day is defined as 8 hours of production). There are six tasks to be completed, with a total task time of 12 minutes. The company should have:

A. 25 workstations

A process with Six Sigma quality is expected to produce how many defects? A. 3.4 defects per million opportunities B. 6 defects per million opportunities C. 66,807 defects per million opportunities D. Zero defects

A. 3.4 defects per million opportunities

Generally, the most expensive transportation mode (from among the following choices) is A. Air. B. Water. C. Truck. D. Rail.

A. Air. Air is generally the most expensive mode of transportation.

"Pass/ fail" and "go/no go" are both examples of: A. Attribute data. B. Decision data. C. Process flow data. D. Variable data.

A. Attribute data. (Pass/fail, go/no go, and good/bad are all attribute data.)

"Quality at the source" suggests quality should be: A. Built into the product. B. Inspected at multiple points. C. Ensured by the quality control lab. D. An inspector's function.

A. Built into the product. (The "quality at the source" philosophy is opposed to the idea that quality comes from thorough inspection after the fact.)

A contemporary process layout in which product "families" with similar processing characteristics are identified and produced on "mini-assembly lines" is referred to as:

A. Cellular layout

Wanda Corp. currently uses a job shop process. It wants to maintain the advantages but increase efficiency. Wanda should investigate:

A. Cellular manufacturing

In a Cost of Quality analysis, "failure costs" refers to: A. Costs associated with defects B. Costs of not achieving goals C. Costs of not considering customer needs D. Costs associated with mistakes

A. Costs associated with defects (Failure costs are costs that arise from defects found in products either before or after the product reaches the customer.)

Effective quality management requires: A. Data B. Workers skilled in statistics and math C. Charismatic leadership D. Willingness to spend time and money measuring output

A. Data

Which process activity is due to other factors (e.g., insufficient operating capacity, lack of material, equipment breakdowns)?

A. Delay

Which of the following forms of capacity can only be determined AFTER the completion of activities?

A. Demonstrated Capacity

Ensuring that all the right people, equipment, and materials arrive on time is especially challenging when using which layout?

A. Fixed-position layout

One function of warehouses or distribution centers is "consolidation." This means that the warehouse/distribution center is used to: A. Group small inbound loads into larger outbound loads. B. Collect products/packaging materials that are sent back for disassembly, reclamation, or disposal. C. Hold inventory to buffer the system from variability in either in supply or demand. D. All of these.

A. Group small inbound loads into larger outbound loads. This is the definition of the consolidation role of warehouses/distribution centers.

Most Processes involve two basic types of flows: A Information flows and physical flows. B. Product flows and decision flows C. Information flows and operator flows. D. Physical flows and documentation flows.

A. Information flows and physical flows.

In determining the number of facility locations, the lowest total cost is typically: A. Not the lowest cost of either transportation or inventory. B. Typically the lowest cost of either transportation or inventory. C. Not related to transportation or inventory. D. Difficult to generalize.

A. Not the lowest cost of either transportation or inventory.

Suppose you were the logistics manager for Way-Off Manufacturing and Marketing, a manufacturer of mosquito repellant. Further suppose that Way-Off markets its products in the entire United States. However, in each geographic region of the country, Way-Off only needs to warehouse products for two months during each year. Way-Off probably would use what type of warehousing? A. Public B. Private C. Contract D. Common

A. Public When warehousing needs for a company are short-term, public warehousing is generally the best option.

Which of the following statements is false? A. Quality dimensions are easy to measure for tangible goods, while they are difficult to measure for services. B. Many dimensions of service quality are similar to the dimensions of quality for tangible goods C. Some firms still have a poor understanding of quality and quality management. D. All of these are false.

A. Quality dimensions are easy to measure for tangible goods, while they are difficult to measure for services.

Choose the service type that is correctly matched with one of its key characteristics

A. Service factory-facilities and equipment represent a larger proportion of total costs

Little's law suggests that the key to increased throughput is:

A. Shorter Flow times

With respect to the following operating service characteristics of the modes of transportation, which combination is CORRECTLY specified? A. Speed: air is best; pipeline is worst. B. Availability: air is best; pipeline is worst. C. Capability: rail is best; air is worst. D. Dependability: pipeline is best; truck is worst.

A. Speed: air is best; pipeline is worst. Air is the fastest mode, pipeline is the slowest. Air is low in availability (location of airports). Water can handle more different types/sizes of shipments than rail. Truck is more dependable than rail or air.

If you were developing a service blueprint, how would you classify "delivery of gasoline to a service station"?

A. Supprt processes

"Product quality" is best explained as: A. The result of both design quality and conformance quality B. Absence of defect C. Customer satisfaction D. Desirable features

A. The result of both design quality and conformance quality (Product quality is a product's fitness for consumption, which is determined by both its' design and conformance quality.)

A company finds the following information about its customers and their locations: The center of gravity is: A. X* = 52.5; Y* = 57.5 B. X* = 58.33; Y* = 65 C. X* = 56.66; Y* = 61.66 D. Cannot be determined.

A. X* = 52.5; Y* = 57.5

Which of the following products is most likely to use an assemble-to-order market orientation?

A. an upholstered sofa

Which of the following types of control charts would be most appropriate if the data that we are dealing with is binary in nature (good/bad, pass/fail, accept/reject)? A. p attribute chart. B. Xbar-R chart. C. Cp/Cpk. D. Histogram.

A. p attribute chart. (A p-Attribute control chart is useful when dealing with binary data.)

Computer-controlled systems that use robots for materials handling activities are:

AS/RS.

Intermodal transportation:

Achieves economies of scale and service availability.

As technology and cultures evolve, the best approach for operations managers is to: Work harder. Adapt their business models. Retire. Make their processes more efficient.

Adapt their business models

As technology and cultures evolve, the best approach for operations managers is to: Work harder. Retire. Adapt their business models. Make their processes more efficient.

Adapt their business models. While some students may argue for the other choices, the theme of the chapter is that technology and culture are important elements of operations that may dictate the need to alter business models.

An example of a short-term capacity decision is: Adding specialized labor. Installing new equipment Adding new facilities Adding low-skilled labor

Adding low-skilled labor

A relationship with a supplier that is characterized by high levels of distrust, little communication, and short-term transactions is called a(n):

Adversarial relationship.

Generally, the most expensive transportation mode (from among the following choices) is

Air

Generally, the most expensive transportation mode (from among the following choices) is Air Truck Rail Water

Air

Your company needs to ship 10 heart pacemakers which cost $10,000 each from Indianapolis IN to Miami FL. Which transportation mode should you use?

Air

Your company needs to ship 10 heart pacemakers that cost $10,000 each from Indianapolis, Indiana, to Miami, Florida. Which transportation mode should you use

Air.

Firms in which of these industries experience no difficulty in balancing supply and demand?

All companies must make difficult decisions in balancing supply and demand

. Which of the following products would be most likely "make to stock"? A. A commercial passenger airplane B. A communications satellite C. A train locomotive D. An automobile

An automobile

Sourcing

An important part of supply management. is the identification, evaluation, selection, and management of suppliers.

Which of the following products is most likely to use an assemble-to-order market orientation? A. An upholstered sofa B. A mobile phone C. Shampoo D. A race car

An upholstered sofa

Sanford Corp. bought new technological systems to inspect the quality of products as they come off the production line. The expense of operating these systems would be an example of which of the following types of quality-related costs?

Appraisal cost

Inventory levels:

Are higher when rail rather than truck transportation is used.

If the cumulative lead time is eight weeks, for a project to be feasible the MPS planning horizon must be:

At least eight weeks.

Where in a project schedule is a good place to plan a buffer?

At the end of the project. Immediately after a highly uncertain task. Where scarce resources are needed. All of these are good places to plan buffers.

Which of the following identifies the most appropriate time to set the metrics when doing a process flow analysis? During the analysis stage At the start of the process When bounding the critical process When recommending the appropriate changes to the process (i.e., the "future state" map)

At the start of the process

"Pass/ fail" and "go/no go" are both examples of:

Attribute data

) If you are evaluating whether a supplier's workforce is receiving fair wages, you are most likely doing a(n): A) Spend analysis. B) Assessment of sustainability. C) ISO 9000 assessment. D) Total cost of ownership assessment.

B

A company has decided that it no longer needs to extensively count and inspect the products it receives from a particular supplier. This suggests that the purchasing company has begun: A) Spend analysis. B) Supplier certification. C) Process simplification. D) Time reduction analysis.

B

A company's supplier for a critical part was selected based on the lowest purchase price. Six months later, the company's warranty costs began to rise because of defective parts. The company has experienced: A) Reduced sustainability. B) Higher total costs of ownership. C) Problems with availability of resources. D) Higher levels of spend analysis

B

All of the following are supply management goals EXCEPT: A) Ensure timely availability of resources. B) Purchase at the lowest price. C) Enhance quality. D) Assess technology and innovation.

B

All of the following can increase supply chain risk except: A) Using a supplier that is having financial difficulty. B) Holding higher levels of inventory of critical materials. C) A company in Canada using suppliers in Thailand. D) Single sourcing.

B

Aspen Corporation has discovered that it uses 10 different suppliers for a particular type of item, has bought 20 different models of that item, and total expenditures last year for the item were $100,000. Aspen most likely is conducting: A) Supplier audits. B) Spend analysis. C) Supplier certification. D) Make or buy analysis

B

Floor cleaning supplies for an elementary school would be considered: A) Leverage. B) Noncritical. C) Strategic. D) Bottleneck.

B

Many companies are automating the procure-to-pay process using: A) Blockchain. B) Robotic process automation (RPA). C) IoT. D) Collaborative planning, forecasting, and replenishment.

B

Outsourcing is often a good choice when: A) There are only a few suppliers. B) A product is in the mature phase of the life cycle. C) Technology is new. D) The product is critical to a firm's competitiveness.

B

Supply management: A) Only focuses on direct materials that are used in a company's products. B) Is essential for an organization to attain its strategic objectives. C) Is only involved with supplier selection. D) Should always strive for the lowest purchase price.

B

When you need to identify a supplier for a new purchase, the FIRST place that you should look is: A) A local trade show. B) The list of your company's current suppliers. C) The list of suppliers that your company has used in the past. D) The websites of industry groups.

B

Which of the following is not uncovered during an analysis of supply market intelligence? A) The number of potential suppliers. B) Which of the company's current purchases are from which supplier. C) The number of other buyers. D) The potential level of supply chain risk.

B

Williams Inc. has acquired software to help manage interactions with its supply base. This suggests that Williams Inc. is involved in: A) Vendor-managed inventory. B) Supplier relationship management. C) Online reverse auctions. D) Supplier auditing.

B

The Rodriguez Manufacturing Co. uses a weighted-point model to evaluate new suppliers. Based on its competitive priorities, quality performance is weighted 50 percent, delivery performance is 20 percent, and cost is 30 percent. On a five-point scale, a supplier is rated 3 on quality, 2 on delivery, and 5 on cost. What is its overall weighted score? A) 10. B) 3.4. C) 33. D) 3.

B The ratings are weighted by the weights and then added together. 0.5 × 3 + 0.2 × 2 + 0.3 × 5 = 3.4

The Smith Manufacturing Co. uses a weighted-point model to evaluate new suppliers. Based on its competitive priorities, quality performance is weighted 60 percent, delivery performance is 25 percent, and cost is 15 percent. On a five-point scale, a supplier is rated 4 on quality, 5 on delivery, and 5 on cost. What is its overall weighted score? A) 14. B) 4.4. C) 4. D) 5.

B The ratings are weighted by the weights and then added together. 0.6 × 4 + 0.25 × 5 + 0.15 × 5 = 4.4.

A bakery has a choice of leasing different types of ovens. Oven A will cost $1,000 per year to lease and operate, plus $0.50 per cake baked. Oven B will cost $3,000 per year to lease and operate, plus $0.40 per cake baked. Find the indifference point at which the annual costs of the two ovens are equal. A) 200 cakes B) 20,000 cakes C) 40,000 cakes D) None of these

B) 20,000 cakes

A company needs to produce 400 units per day (the day is defined as 12 hours of production). There are three tasks to be completed, with a total task time for all three tasks of 6 minutes. The company should have: A) 3 workstations B) 4 workstations C) 5 workstations D) 2 workstations

B) 4 workstations

You realize that you need a tool for recording all the move distances, process flow symbols, people involved, value add designations, and time required for each step to get an overall grasp for where waste exists in the process. The best tool for this would be: A) A value stream map B) A process flow table C) An assembly process chart D) A physical layout diagram

B) A process flow table

If the cumulative lead time is 8 weeks, to be feasible the MPS planning horizon must be: A) Exactly 8 weeks B) At least 8 weeks C) Less than 8 weeks D) At least 16 weeks

B) At least 8 weeks

Which of the following is NOT one of the costs considered in aggregate production planning? A) Subcontracting cost B) Capital equipment cost C) Inventory cost D) Firing (layoff) cost

B) Capital equipment cost

Significant changes in an MPS quantity several weeks into the planning horizon can: A) Make the MRP more accurate B) Cause inconsistencies in the MRP C) Reduce stockouts D) Increase safety stock

B) Cause inconsistencies in the MRP

In its promotional material for bed linens, Scarsdale Corporation states that all sheets and pillowcases sold to customers should have and actually do have a thread count of exactly 600 threads per inch. Scarsdale is apparently emphasizing: A) Design quality B) Conformance quality C) Product quality D) Total quality management

B) Conformance quality

Zanda Company is looking for a requirements planning system to help plan replenishment of its finished goods throughout its network of distribution centers. The type of system Zanda should consider is known as: A) CRP B) DRP C) MRP D) ERP

B) DRP

You walk into a hair stylist shop. All stylists are busy and you sit in the waiting area. You are in which of the following process activities? A) Operations B) Delay C) Storage D) You are not in a process activity

B) Delay

What decisions do supply managers make that potentially impact quality? A) Design of manufacturing and service processes B) Design of contracts and associated incentives and penalties C) Design of packaging D) Design of product prototyping procedures

B) Design of contracts and associated incentives and penalties

If a product includes features that customers care most about, we would say that it has high: A) Conformance quality B) Design quality C) Reliability D) Aesthetics

B) Design quality

If a company spends more on prevention, what would be the expected impact on other costs of quality? A) Appraisal costs will go up B) Failure costs will go down C) Failure costs will go up D) Other costs will be unaffected

B) Failure costs will go down

Systems that combine automated machines, robots, and material handling systems and are controlled by a single computer are: A) Transportation management systems (TMS). B) Flexible manufacturing systems (FMS). C) Manufacturing executive systems (EMS). D) Warehouse management systems (WMS).

B) Flexible manufacturing systems (FMS).

If a company strongly prefers that its aggregate output plan be closer to a level plan than a chase plan, this implies that it is concerned about minimizing: A) Inventory carrying costs B) Hiring and layoff costs C) Cost of subcontracting D) Both A and C

B) Hiring and layoff costs

You, as a supply chain analyst, have been approached by the vice president of supply chain operations to come up with a recommendation for improving the throughput time for the manufacture of Widget A. You quickly realize that you need a process map before you proceed. What is the first step you should undertake to map the process? A) Draw a detailed schematic of the inputs and outputs of the process in question B) Identify the desired outcomes of the process C) Analyze the process and prioritize opportunities for improvement D) Document the existing process (the "current state" map).

B) Identify the desired outcomes of the process

Government regulations belong to which of the following elements of a process? A) Activities B) Inputs/outputs/feedback C) Process structure D) Management policies

B) Inputs/outputs/feedback

A short-term, highly focused effort for improving a process is known as a: A) Jidoka Event B) Kaizen Event C) Chaebol Event D) Poka-yoke Event

B) Kaizen Event

A bill-of-materials file contains: A) Invoices for materials that must be paid in this period B) Lists of materials required to produce end item products C) Accounts payable for materials D) A and B

B) Lists of materials required to produce end item products

The TQM view of organizational structure states that A) Employees must support management decisions B) Management must support employees C) Customers want quality products D) Middle management must support and enforce top management decisions

B) Management must support employees

We are experiencing a problem where a process keeps shifting over time. If we were to use a histogram to collect and display the resulting data, what type of shape would it most likely display? A) Bell (normal) distribution B) Plateau-shaped distribution C) Bimodal distribution D) Skewed distribution

B) Plateau-shaped distribution

Three processes have the following costs: Process A has a fixed cost of $2,000 and variable cost of $3.00/unit. Process B fixed cost is $4,000 and variable cost is $2.60/unit. Process C fixed cost is $8,000 and variable cost is $2.40/unit. If the projected total demand is for 6,000 units, which process should be used? A) Process A B) Process B C) Process C D) None of the processes work

B) Process B

A common goal when designing a functional layout is to: A) Ensure materials arrive on schedule B) Reduce the time and cost of moving people and materials between departments C) Avoid downtime at any workstation D) Balance the flow of materials through the process

B) Reduce the time and cost of moving people and materials between departments

Which process type is not correctly matched with its descriptive characteristic? A) Project-high complexity B) Repetitive process-customized products C) Job shop-high flexibility D) Continuous flow-low-skilled workers

B) Repetitive process-customized products

You observe three different people doing the same task using three different approaches (with differing levels of quality, output, and lead time). Which of the following disposition options is most appropriate for this situation? A) Keep B) Rethink C) Combine D) Elimate

B) Rethink

Each month the sales and operations team at Johnson Company meets to develop plans for each of the next six months. This process is known as: A) Collaborative planning on forecasting B) Rolling planning horizons C) Unconstrained planning D) Continuous planning

B) Rolling planning horizons

Joe's Quick Oil Change shop is interested in improving its ability to satisfy customers. The most appropriate mapping tool for a company such as this is called: A) Service architecture B) Service blueprinting C) Service diagramming D) Service mapping

B) Service blueprinting

Implementation of a Six Sigma program typically involves: A) Applying for the Malcolm Baldrige Award B) Setting up training at various levels including green belt and black belt training C) A cost of quality analysis D) None of these are usually involved in implementing Six Sigma

B) Setting up training at various levels including green belt and black belt training

Constant changing of the master schedule and the resulting changes in the requirements for components is referred to as: A) Master schedule dynamics B) System nervousness C) Bad planning D) MRP failure

B) System nervousness

Decisions being made about the aggregate production plans represent what type of planning? A) Strategic level planning B) Tactical planning C) Detailed operational planning D) Long-term planning

B) Tactical planning

A bottleneck activity in a process is generally the activity with: A) The smallest number of resources devoted to it B) The least capacity C) No delays in front of it D) The highest cost

B) The least capacity

What is the primary objective of line balancing? A) To minimize the travel time between workstations B) To match output rates with actual demand C) To minimize materials handling costs D) To group similar activities together

B) To match output rates with actual demand

The specifications for the diameter of a molded part are 10 mm ± 0.5 mm. The actual average and standard deviation from 250 parts sampled is 10.1 mm and 0.1 mm, respectively. What is the process capability as measured by Cp? Multiple Choice A. 0.84 B. 1.67 C. 0.60 D. 1.00 E. 1.01

B. 1.67 (Cp = (10.5 − 9.5) ÷ (6 × 0.1) = 1 ÷ 0.6 = 1.67)

A bakery has a choice of leasing different types of ovens. Oven A will cost $1000 per year to lease and operate, plus $0.50 per cake baked. Oven B will cost $3000 per year to lease and operate, plus $0.40 per cake baked. Fine the indifference point at which the annual costs of the two ovens are equal.

B. 20000 cakes

A company needs to produce 400 units per day(the day is defined as 12 hours of production.) There are three tasks to completed, with a total stark time for all three tasks of 6 minutes. The company should have:

B. 4 workstations

The Plan-Do-Check-Act cycle is best defined as: A. A rigorous procedure for strategic planning. B. A method used to guide problem identification and solution. C. A method first proposed by Joseph Juran. D. All of these statements apply to the definition.

B. A method used to guide problem identification and solution. (PDCA is a simple problem-solving method, also called the "Deming wheel.")

"Quality management" is: A. A measure of how well managers achieve goals B. An approach that develops culture and tools with a focus on quality C. Six sigma D. The achievement of all dimensions of quality

B. An approach that develops culture and tools with a focus on quality (Quality management establishes an organizational focus on quality—developing an organizational culture of quality and equipping employees with appropriate quality management tools.)

Sanford Corp. bought new technological systems to inspect the quality of products as they come off the production line. The expense of operating these systems would be an example of which of the following types of quality-related costs? A. Internal failure cost B. Appraisal cost C. External failure cost D. Prevention cost

B. Appraisal cost (Inspection is an appraisal cost.)

In its promotional material for bed linens, Scarsdale Corporation states that all sheets and pillowcases sold to customers should have and actually do have a thread count of exactly 600 threads per inch. Scarsdale is apparently emphasizing: A. Design quality. B. Conformance quality. C. Product quality. D. Reliability. E. Total quality management.

B. Conformance quality.

Zanda Corporation's CEO watched in amazement and frustration as workers handled and loaded individual products onto trucks. She felt handling the individual items was very cumbersome. She should investigate the possibility of: A. Using RFID. B. Containerization. C. AS/RS. D. None of these.

B. Containerization. Containerization creates larger containers out of smaller units to facilitate handling.

Richard's Rollerskate Company relies on Simon's Trucking to transport all of their products. Simon's Trucking is the only shipping company for RRC, and all products are shipped in trucks that look like giant roller skates with Richard's Rollerskate branding on the trucks. Richard's Rollerskate is likely using which carrier type? A. Common carrier. B. Contract carrier. C. Private carrier. D. None of these.

B. Contract carrier. When the carrier takes on the identity of a shipper, it is typically a contract carrier.

If a product includes features that customers care most about, we would say that it has high: A. Conformance quality. B. Design quality. C. Reliability. D. Aesthetics.

B. Design quality. (Design quality occurs when product features meet customers' desires and needs.)

If a company spends more on prevention, what would be the expected impact on other costs of quality? A. Appraisal costs will go up. B. Failure costs will go down. C. Failure costs will go up. D. Other costs will be unaffected.

B. Failure costs will go down. (Appraisal costs are a choice, independent of prevention costs, whereas failure costs are likely to go down with well-framed prevention efforts. Some costs need to be impacted, otherwise the investment in prevention cost is not of any value.)

Systems that combine automated machines, robots, and material handling systems and are controlled by a single computer are:

B. Flexible manufacturing systems (FMS)

Government regulations belong to which of the following elements of a process?

B. Inputs/outputs/feedback

A major disadvantage of private warehousing is: A. It offers less control than contract warehousing. B. It requires more investment than contract warehousing. C. It has less flexibility in operating policies than contract warehousing. D. All of these are disadvantages of private warehousing.

B. It requires more investment than contract warehousing. The disadvantage is the investment required. Private warehousing provides more control and operational flexibility

A short term, highly focused effort for improving a process is known as a:

B. Kaizen Event

The TQM view of organizational structure states that: A. Employees must support management decisions. B. Management must support employees. C. Customers want quality products. D. Middle management must support and enforce top management decisions.

B. Management must support employees.

Two dimensions of product quality are hard to measure objectively and are therefore subject to subjective assessment. One of those subjective dimensions is: A. Reliability B. Perceived quality C. Support D. Features

B. Perceived quality (Perceived quality is a subjective assessment based upon image, advertising, reputation, or other information related to the product's attributes. The other subjective dimension of product quality is aesthetics.)

Ted wants to hire more inspection personnel to ensure that all products shipped are free of defects. What TQM core value would this action violate? A. Workers are more valuable than managers B. Prevention is better than rework C. Better quality leads to long term success D. A PDCA cycle should always be followed

B. Prevention is better than rework

Three processes have the following costs: Process A has a fixed cost of $2000 and variable cost of $3.00/unit. Process B fixed is $4000 and variable cost is $2.60/unit. Process C fixed cost is $8000 and variable cost is $2.40/unit. If the projected total demand is for 6000 units, which process should be used?

B. Process B

A common goal when designing a functional layout is to:

B. Reduct the time and cost of moving people and materials between departments

Which process type is not correctly matched with its descriptive characteristic?

B. Repetitive process-customized products

An approach that analyzes the interface between customers and service processes is called:

B. Service blueprinting

One implication of increased utilization in a process is that:

B. Waiting time typically will go up

Miller Corp. can make three individual deliveries to three different customers at a transportation cost of $600 each or can consolidate them into one shipment with a transportation cost of $1,200 and a stop-off charge of $100 per stop. Should Miller Corp. consolidate the shipments? A. Yes, it will save $100. B. Yes, it will save $300. C. No, it will increase cost by $100. D. It doesn't matter based on cost.

B. Yes, it will save $300. Nonconsolidated cost = $600(3) = $1,800. Consolidated cost = $1,200 + $100(3) = $1,500. Consolidation saves $300

Why is product quality sometimes poorly defined in a firm?

Both B and C are correct.

Moderate- to high-risk purchases with low to moderate levels of spend are categorized as:

Bottleneck

Moderate- to high-risk purchases with low to moderate levels of spend are categorized as:

Bottleneck.

Moderate- to high-risk purchases with low to moderate levels of spend are categorized as: Noncritical. Leverage. Bottleneck. Strategic.

Bottleneck. Moderate- to high-risk purchases with low to moderate levels of spend are categorized as bottleneck, as shown in Figure 10-3.

Splitting shipments into individual orders and arranging them for local delivery to customers is:

Break-bulk

Splitting shipments into individual orders and arranging them for local delivery to customers is:

Break-bulk.

Stainless steel is an important raw material for an appliance company. Steel accounts for a high level of spend and is critical to customer satisfaction. Further, only three suppliers worldwide can produce steel to meet the company's quality standards. Which is appropriate in this situation?

Build collaborative partnerships with suppliers.

Stainless steel is an important raw material for an appliance company. Steel accounts for a high level of spend and is critical to customer satisfaction. Further, only three suppliers worldwide can produce steel to meet the company's quality standards. Which is appropriate in this situation? Using electronic catalogs for steel. Finding substitute materials. Use competition to select suppliers. Build collaborative partnerships with suppliers.

Build collaborative partnerships with suppliers. The high value of spend and the high level of supply risk suggests that this is strategic situation. Building collaborative partnerships is appropriate in this situation.

"Quality at the source" suggests quality should be:

Built into the product.

Which of the following statements best describes the role of the business model in today's environment?

Business models must be regarded as dynamic.

A company requires its suppliers to make its products in at least three different locations around the world. Which supply management goal does this BEST accomplish? A) Fostering sustainability. B) Reducing total costs. C) Identifying, assessing, and mitigating supply chain risk. D) Accessing technology and innovation.

C

A company that had been using single sourcing decided to add a second supplier for one of its key raw materials. Which supply management goal does this BEST accomplish? A) Fostering sustainability. B) Reducing total costs. C) Identifying, assessing, and mitigating supply chain risk. D) Accessing technology and innovation.

C

A comprehensive system, facilitated by software, that manages the firm's interactions with its supply base is: A) Supplier certification. B) Supplier scorecard. C) Supplier relationship management. D) Collaborative planning, forecasting, and replenishment.

C

A legally binding document that signals to a supplier that goods and services are needed is a(n): A) Purchase requisition. B) Electronic data interchange. C) Purchase order. D) Supplier relationship management document.

C

An online auction used for sourcing: A) Typically drives prices up as suppliers compete against each other. B) Is used in situations similar to negotiation. C) Can hurt supplier relationships. D) Can only consider price.

C

Data on the supply market's structure, price forecasts, and supply chain risk are assessed during a/an: A) Spend analysis. B) Insourcing/outsourcing decision. C) Analysis of supply market intelligence. D) Supplier certification process.

C

Determining the right number of suppliers that a company should use is: A) Spend analysis. B) Strategic sourcing. C) Supply base optimization. D) Supply chain resilience.

C

Negotiation is typically used when: A) Price is the most important factor. B) There are many equally qualified suppliers who are willing to compete. C) Early supplier involvement is needed in new product development. D) There are standard product specifications that are clear and complete.

C

Outsourcing: A) Should be used for an organization's core activities. B) Always uses suppliers from developing countries. C) Can reduce a company's capital requirements D) Is only done for services

C

The type and level of assessment of suppliers during the selection process: A) Should be the same for all of a company's purchases. B) Is always done by cross-functional teams. C) Depends upon factors such as the level of spend and type of relationship desired. D) Is normally very extensive for noncritical purchases.

C

Which activity at a university is the least likely to be outsourced? A) Operation of the bookstore. B) Operation of the residence halls. C) Classroom instruction. D) Information technology support services

C

Which of the following is NOT a benefit of outsourcing? A) Greater flexibility to change technology B) Better access to market information C) Lower supply management costs D) Less capital needed for investment

C

Zanda Corp. had outsourced its production to a company located in Asia. Recently it decided to continue to outsource but bring the production back to a company located in the United States. This decision was likely made after Zanda: A) Conducted a spend analysis. B) Conducted a make or buy analysis. C) Examined total cost of ownership. D) Developed supplier certification processes.

C

Zanda Corp. is considering trying to develop a full partnership with a particular supplier. This suggests that the item Zanda buys from the supplier is: A) A noncritical part. B) A "bottleneck" item. C) A strategically important part. D) An item appropriate for a "leverage" strategy.

C

Advanced Manufacturing Company is evaluating two suppliers for a component sourcing. After much internal discussion, AMC's management has determined that the critical factors in choosing suppliers are: quality, delivery, price, and service. Further, management has rated the importance of these factors as 0.4, 0.3, 0.2, and 0.1, respectively. On a scale of 1 to 5, Supplier A is rated at 5, 3, 3, and 3, respectively. Supplier B's ratings are 4, 4, 3, and 4. Which supplier has the better-weighted score? A) Supplier A B) Supplier B C) Neither, they both have the same weighted score.

C Supplier A's score = .4(5) + .3(3) + .2(3) + .1(3) = 3.8. Supplier B's score = .4(4) + .3(4) + .2(3) + .1(4) = 3.8

Which of the following is true concerning sales and operations planning? A) Once the plan is finalized, it should not be changed during the planning period B) There is a specific set of steps all firms should follow in the sales and operations planning process. C) A benefit of the process is that the firm should achieve high service levels with lower inventory D) All of these are true

C) A benefit of the process is that the firm should achieve high service levels with lower inventory

Which of the following conditions does a "critical" process satisfy? A) Most steps (thus consuming the most time) B) The one that people complain about the most C) A process that is visible to the key customers D) The first process that we encounter in the system

C) A process that is visible to the key customers

The positioning and replenishment of finished goods inventories at the retail level can be determined using: A) MRP B) CRP C) DRP D) RFID

C) DRP

Which of the following activities is most likely to be overlooked when doing a process flow analysis? A) Operations B) Inspections C) Delays D) Storages

C) Delays

The objective of statistical process control (SPC) is to: A) Determine whether a batch of raw materials should be accepted or rejected B) Determine whether a batch of finished goods should be accepted or rejected C) Detect assignable cause variations versus normal random variations in the process D) Identify variable versus attribute measures

C) Detect assignable cause variations versus normal random variations in the process

Which of the following statements about economies of scale is NOT true? A) One reason economies of scale occur is because fixed costs can be spread over more units of production as output increases B) One reason economies of scale occur is because employees become more efficient as volume increases C) Economies of scale refers to the fact that as volume increases, total cost of production decreases. D) Economies of scale may not exist at all levels of production

C) Economies of scale refers to the fact that as volume increases, total cost of production decreases.

The focus of an aggregate production plan, in general, is on all of the following EXCEPT: A) The intermediate-term future rather than the very short-term future B) Product lines rather than specific items C) Facilities and capital equipment rather than labor and inventory D) Inventory levels rather than new plants

C) Facilities and capital equipment rather than labor and inventory

Zanda Corp. has determined that it has too many products returned from customers. Zanda is interested in determining what factors or elements could contribute to the large number of returns. Which of the following quality tools would Zanda use to achieve this objective? A) Process flowchart B) Pareto analysis C) Fishbone diagram D) Run diagram

C) Fishbone diagram

John Jones, CEO of Joes Corp., is unhappy because each product his company makes takes a unique route through the facility, so processing times tend to be high. His company most likely has a: A) Fixed-position layout B) Sequential action layout C) Functional layout D) Product layout

C) Functional layout

Zanda Corp. and Jones Corp. are identical in every way (products produced, costs, demand, etc.) except for one. Zanda uses a level production plan while Jones prefers a chase production plan. Which of the following is most likely to be true? A) Jones will have higher investment in plant and equipment B) Jones will have higher hiring and firing costs C) Jones will have lower inventory carrying costs D) All of the options are true

C) Jones will have lower inventory carrying costs

End item A is made by assembling 2 of part B and 3 of part C. Part C is made from 2 of component D and 2 of component E. Component D is considered to be a: A) Level 0 B) Level 1 input C) Level 2 input D) Level 3 input

C) Level 2 input

Which of the following identifies the ultimate goal of process mapping? A) Reduce steps in the process. B) Reduce costs created by the process. C) Make the desired outcome inevitable D) Facilitate the identification and reduction of system complexity

C) Make the desired outcome inevitable

Which of the following actions will NOT increase output? A) Increasing capacity through physical additions B) Outsourcing C) Managing the flow of work into the system D) Changing the processes to eliminate unnecessary steps

C) Managing the flow of work into the system

Using technology, such as self-checkouts in grocery stores, to enable customers to complete the service delivery and transaction themselves is most commonly used for: A) Service factories B) Service shops C) Mass services D) Professional services

C) Mass services

The principles of process improvement suggest that an effective, efficient process: A) Maximizes the number of sequential activities and minimizes the number of parallel ones B) Eliminates all non-value-added steps C) Minimizes the number of entry points of a workpiece into the process D) Minimizes reliance on departments outside of manufacturing

C) Minimizes the number of entry points of a workpiece into the process

How does aggregate planning for services differ from aggregate production planning for products? A) There is no difference B) Demand for products typically is stated as the number of hours of labor required, whereas demand for services is generally stated as the number of units of service desired. C) Most service plans are based primarily on labor requirements D) Service plans make extensive use of inventory to meet demand

C) Most service plans are based primarily on labor requirements

MRP is a process that would be most applicable in which situation? A) Planning requirements for repair parts for production equipment in a manufacturing plant B) Planning requirements for safety stocks of finished goods in a distribution center C) Planning requirements for bicycle seats in a bicycle production plant D) A and C

C) Planning requirements for bicycle seats in a bicycle production plant

Xanadu Inc. decided to increase the training received by new employees. The expense of this training is an example of which of the following costs of quality: A) Appraisal costs B) External failure cost C) Prevention costs D) Quality control costs

C) Prevention costs

Kaizen Events are most appropriate for which type of operations problem? A) Customer-based B) Metrics-based C) Process-based D) Product-based

C) Process-based

Which of the following categories of causes would be most appropriate for a Marriott (an international hotel chain) or McDonalds (an international fast food chain) or the local hair stylist? A) People, Machine/Equipment, Methods/Processes, Materials B) Machine, Method, Material, Manpower/Mindpower, Measurement, Mother Nature C) Product, Price, Place, Promotion, People/personnel, Process, Physical evidence D) People, Price, Machine, Materials, Mother Nature

C) Product, Price, Place, Promotion, People/personnel, Process, Physical evidence

Which of the following is NOT a core value of TQM? A) Prevention is better than inspection B) Problem solving should occur at the lowest organizational levels possible C) Quality assurance managers should have the most power in the organization D) Product quality should be viewed holistically

C) Quality assurance managers should have the most power in the organization

Companies with a make-to-stock orientation are most likely to use which of the following process structures? A) Mass customization B) Job shop C) Repetitive process D) They are likely to use any one of these process structures

C) Repetitive process

Planning for which of the following products would see the LEAST benefit from the use of an MRP system? A) Microwave ovens B) Mobile phones C) Shampoo D) Power lawnmowers

C) Shampoo

Phil Bord is a CEO for a large auto manufacturer and is interested in improving the product quality. Phil had overheard his friend mention Six Sigma. Considering that Phil has no idea what Six Sigma is, what are some key points to help Phil out? A) Six sigma quality means that he would have exactly half as many quality defects as he would have with three sigma quality. B) To achieve truly world-class quality and performance, we need to focus on individual employee job functions. C) Six sigma quality is the result of a well-defined and structured process D) All of these

C) Six sigma quality is the result of a well-defined and structured process

The following sequence shows four operations for a computer chip assembly process and the effective capacity of each. Which step is the bottleneck? Step 1: 500 chips/hour Step 2: 250 chips/hour Step 3: 200 chips/hour Step 4: 550 chips/hour A) Step 1 B) Step 2 C) Step 3 D) Step 4

C) Step 3

Adam Smith described his company's process as follows: "Under ideal conditions we can produce 10,000 units a day. However, our normal production is 6,000 units a day. Today, we actually produced 8,000 units." Which of the following is true? A) Utilization is 60 percent of maximum capacity B) Yield is 133 percent of effective capacity C) Utilization was 133 percent of effective capacity D) All of these are true

C) Utilization was 133 percent of effective capacity

Under what conditions would we be required to recalculate the parameters for an X-bar-R chart? A) When the first sample mean crosses either the UCL or LCL B) When there is hugging observed in the data C) When we change the production process D) When dealing with a two-shift operation

C) When we change the production process

Zanda Corp. and Jones Corp. are identical in every way (products produced, costs, demand, etc.) except for one. Zanda uses a level production plan while Jones prefers a chase production plan. Which of the following is most likely to be true? A) Zanda will have higher investment in plant and equipment. B) Zanda will have higher hiring and firing costs C) Zanda will have higher inventory carrying costs D) Zanda will have lower total production costs

C) Zanda will have higher inventory carrying costs

Sample means and ranges were obtained for five samples of 10 units per sample from a production process. Assume the process was considered to be in control during the period these samples were collected. The results are as follows: Sample 1, 2, 3, 4, 5 Mean 10.58, 9.97, 9.99, 10.23, 10.23 Range0.23, 0.25, 0.214, 0.28, 0.28 What is the central line of the X chart for this process A. 10.00 B. 10.05 C. 10.20 D. 9.45 E. 9.75

C. 10.20 (The central line of the X chart is the mean value of the sample means = 10.20)

. Which of the following is the best example of a service shop? A. A fast food restaurant such as McDonald's B. An electrical utility company C. A computer repair shop D. A dentist

C. A computer repair shop

Suppose you are shipping 1,000 pounds of product to a customer location that is 500 miles away from you. The customer calls you and increases the order size to 10,000 pounds. Which of the following is NOT LIKELY to occur as a result of this change? A. The cost per pound of transportation for the order will decrease. B. The total cost of transportation for the order will increase. C. Both of these are likely to occur.

C. Both of these are likely to occur. This consolidation is likely to both decrease the cost per pound of transportation and increase the total cost.

With respect to the following operating service characteristics of the modes of transportation, which combination is INCORRECTLY specified? A. Speed: air is best; pipeline is worst. B. Availability: truck is best; pipeline is worst. C. Capability: pipeline is best; water is worst. D. Dependability: pipeline is best; air is worst.

C. Capability: pipeline is best; water is worst. Only C is incorrect. For capability, pipeline is worst, water is best.

The objective of statistical process control (SPC) is to: A. Determine whether a batch of raw materials should be accepted or rejected. B. Determine whether a batch of finished goods should be accepted or rejected. C. Detect assignable cause variations versus normal random variations in the process. D. Identify variable versus attribute measures.

C. Detect assignable cause variations versus normal random variations in the process. (Statistical process control is used to identify variations that arise from other than normal variations in a process. The terms "statistical process control" and "process control charts" convey the same concept.)

The center-of gravity method of determining a facility location: A. Requires that demand be estimated in terms of the dollar sales revenue in each market. B. Requires that cost be measured in dollars of transportation cost. C. Does not require that qualitative factors be considered in location decisions. D. All of these.

C. Does not require that qualitative factors be considered in location decisions.

Which of the following statements about economies of scale is NOT true?

C. Economies of scale refers to the fact that as volume increases, total cost of production decreases

Zanda Corp. had established a production plant in Thailand. Recently it decided to close the plant and move production to Mexico. This decision was likely made after Zanda: A. Conducted a spend analysis. B. Conducted a make-or-buy analysis. C. Examined total landed cost. D. Evaluated consolidation strategies.

C. Examined total landed cost. Total landed cost requires examination of all product- and logistics-related costs. This type of examination may result in decisions regarding the lowest-cost place for manufacturing

Zanda Corp. has determined that it has too many products returned from customers. Zanda is interested in determining what factors or elements could contribute to the large number of returns. Which of the following quality tools would Zanda use to achieve this objective? A. Process flowchart B. Pareto analysis C. Fishbone diagram D. Run diagram E. Scatter diagram

C. Fishbone diagram (The fishbone diagram displays the relationships between observed effects and their possible causes.)

John Jones, CEO of Joes Corp., is unhappy because each product his company makes takes a unique route through the facility, so processing times tend to be high. His company most likely has a:

C. Functional layout

An automated process can be compared to a more labor-intensive process using:

C. Indifference analysis

Which of the following actions will NOT increase output?

C. Managing the flow of work into the system

Zanda Corp. wants to have a process that has cost advantages similar to continuous or repetitive processes, but wants to produce greater variety than those processes normally allow. Zanda should consider:

C. Mass customization

A process has been operating very capably with all of its' output falling between the UCL and the LCL. None-the-less, there is some level of variation in the measurements of the output of the process. What is the source of this variation? A.Less experienced operators B. Inconsistent raw materials C. Natural variation D. Equipment wear and vibration

C. Natural variation (Processes have natural variation)

Zanda Corp. can make three individual deliveries to three different customers at a cost of $500 each or can consolidate them into one shipment with a cost of $1,300 and a stop-off charge of $100 per stop. Should Zanda do this? A. Yes, it will save $100. B. Yes, it is simpler to do. C. No, it will increase cost by $100. D. It doesn't matter based on cost.

C. No, it will increase cost by $100. $1,300 + 3($100) = 1,600, an increase in cost of $100.

You are involved in the implementation of a quality improvement program. Your immediate goal is to identify what problems you should focus on (ideally you want to focus on those that give you the biggest bang for the buck). Which of the following quality control procedures is the most appropriate? A. Fishbone diagrams B. Process flow charts C. Pareto analysis D. Histograms

C. Pareto analysis (Pareto analysis examines the relative intensity of problems and aids in determining where remedial action will have the greatest impact.)

Jones Company learned that several other shippers in its hometown had customers located in the same markets that it serves. With this information, Jones should investigate the possibility of: A. Market area consolidation. B. Scheduled delivery consolidation. C. Pooled delivery consolidation. D. All of these.

C. Pooled delivery consolidation. Because other shippers are involved in this question, pooled delivery consolidation is the most appropriate answer.

Which of the following categories of causes would be most appropriate for a Marriott (an international hotel chain) or McDonalds (an international fast food chain) or the local hair stylist? A. People, Machine/Equipment, Methods/Processes, Materials B. Machine, Method, Material, Manpower/Mindpower, Measurement, Mother Nature C. Product, Price, Place, Promotion, People/personnel, Process, Physical evidence D. People, Price, Machine, Materials, Mother Nature

C. Product, Price, Place, Promotion, People/personnel, Process, Physical evidence

Which of the following is NOT a core value of TQM? A. Prevention is better than inspection. B. Problem solving should occur at the lowest organizational levels possible. C. Quality assurance managers should have the most power in the organization. D. Product quality should be viewed holistically.

C. Quality assurance managers should have the most power in the organization.

Companies with a make-to-stock orientation are most likely to use which of the following process structures?

C. Repetitive process

Which of the following is NOT a basic activity type in processes? A. Delay B.Operation C. Reporting D.Storage E. Decision

C. Reporting

Jones Company has decided to limit deliveries to selected areas to specific days of the week to obtain transportation savings. This is an example of: A. Market consolidation. B. Pooled delivery consolidation. C. Scheduled delivery consolidation. D. Transportation consolidation. E. Poor planning.

C. Scheduled delivery consolidation This is the definition of scheduled delivery consolidation

The following sequence shows four operations for a computer chip assembly process and the effective capacity of each. Which step of the bottleneck? Step 1: 500 chips/hour Step 2: 250 chips/hour Step 3: 200 chips/hour Step 4 550 chips/hour

C. Step 3

Which of the following best describes the concept of total landed cost? A. The sum of all logistics costs related to a product. B. The lowest logistics cost for a product. C. The sum of all product- and logistics-related costs. D. None of these. Total landed cost is the sum of all product- and logistics-related costs.

C. The sum of all product- and logistics-related costs.

Adam Smith described his company"s process as follows: "Under ideal conditions we can produce 10000 units a day. However, our normal production is 6000 units a day. Today, we actually produced 8000 units." Which of the following is true? A. Utilization is 60 percent of maximum capacity B. Yield is 133 percent of effective capacity C. Utilization was 133 percent of effective capacity D. All of these are true

C. Utilization was 133 percent of effective capacity

Under what conditions would we be required to recalculate the parameters for an X-bar-R chart? A. When the first sample mean crosses either the UCL or LCL. B. When there is hugging observed in the data. C. When we change the production process. D. When dealing with a two-shift operation.

C. When we change the production process. (The x-bar and R charts will need to be modified when the production process changes.)

Which of the following would not be considered to be an automating technology?

CPFR

The forward pass in a critical path algorithm involves:

Calculating the earliest possible start and finish times for each task.

Sustainability: Focuses primarily on increasing environmental compliance. Can lower a company's total cost. Does not include worker safety. Focuses only on social issues.

Can lower a company's total cost. Sustainability can reduce a company's total costs by reducing waste and avoiding problems.

Outsourcing:

Can reduce a company's capital requirements

Outsourcing: Should be used for an organization's core activities. Always uses suppliers from developing countries. Can reduce a company's capital requirements Is only done for services.

Can reduce a company's capital requirements Outsourcing reduces a company's capital requirements because it does not need to invest in equipment or facilities to make the sourced product.

With respect to the following operating service characteristics of the modes transportation, which combination is incorrectly specified? Capability: pipeline is best; water is worst Speed: air is best; pipeline is worst Dependability: pipeline is best; air is worst Availability: truck is best; pipeline is worst

Capability: pipeline is best, water is worst

With respect to the following operating service characteristics of the modes of transportation, which combination is INCORRECTLY specified?

Capability: pipeline is best; water is worst.

The specifications for the diameter of a molded part are 10 mm ± 0.5 mm. The actual average and standard deviation from 250 parts sampled is 10.1 mm and 0.1 mm, respectively. The process can be characterized as:

Capable.

Which of the following is NOT one of the costs considered in aggregate production planning?

Capital equipment cost.

A specific analysis of the total greenhouse gases caused directly or indirectly by a product is called: Carbon footprinting. Life cycle analysis. Life cycle waste assessment matrix. Cap and trade.

Carbon footprinting.

A specific analysis of the total greenhouse gases caused directly or indirectly by a product is called: Carbon footprinting. Life cycle analysis. Life cycle waste assessment matrix. Cap and trade.

Carbon footprinting. Carbon footprinting is an analysis of the total greenhouse gases caused directly or indirectly by a product. Life cycle analysis and LCWAM are more extensive than carbon footprinting.

Project-process matrix

Categorizes processes into structures based on output volume and variety. (Project, Job shop, Batch, Repetitive process, Continuous process)

Service process matrix

Categorizes service processes based upon the degree of customization/customer interaction and labor/capital intensity (professional service, service factory, service shop, mass service)

Picture Management has decided that they must do something with the inability of the taillight assemblies to pass the quality specifications. Yet currently they do not know what areas they should look at. To help management in dealing with this issue, which of the following quality control tools would be most appropriate?

Cause-and-effect diagrams

A contemporary process layout in which product "families" with similar processing characteristics are identified and produced on "mini-assembly lines" is referred to as:

Cellular Layout

What contemporary process layout in which product "families" with similar processing characteristics are identified and produced on "mini-assembly lines" is referred to as

Cellular Layout

Wanda Corp. currently uses a job shop process. It wants to maintain the advantages of this but increase efficiency. Wanda should investigate

Cellular Manufacturing

Wanda Corp. currently uses a job shop process. It wants to maintain the advantages of this but increase efficiency. Wanda should investigate:

Cellular Manufacturing

Which of the following is NOT a principle of process improvement? Produce at the rate of customer demand Sequence activities to minimize setups, distance, or other transition costs Change process design to facilitate product improvements All of these are principles of process improvement

Change process design to facilitate product improvements

Which activity is a university least likely to outsource? Operation of the bookstore. Operation of the residence halls. Classroom instruction. Information technology support services.

Classroom instruction. Classroom instruction is a core activity for a university and therefore is least likely to be outsourced

A growing emphasis on involving supply chain partners in product/process innovation is known as:

Codevelopment

Flexible manufacturing systems (FMS)

Combine automated machines, robots, and material handling systems that are all controlled by a single computer (Personalized M&Ms)

Cross-docking

Combines break-bulk and consolidation activities.

Cross-docking:

Combines break-bulk and consolidation activities.

what does Cross-docking do?

Combines break-bulk and consolidation activities.

integrative(综合的, 一体化的) technologies

Combines data management, communications, decision support, and processing capabilities

Jones Company promised a customer that the customer would receive at least 98 percent of all items ordered. In fact, the customer received 95 percent of the items. This is an example of which "gap" in the customer satisfaction model?

Communications Gap

Which of the following would NOT be a step in conducting an insourcing/outsourcing analysis?

Conducting a detailed internal audit of purchasing practices

Which of the following would NOT be a step in conducting an insourcing/outsourcing analysis? Assessing quantitative costs of outsourcing Evaluating new suppliers who could make the SKU Assessing the relationship of the product to the firm's core competencies Conducting a detailed internal audit of purchasing practices

Conducting a detailed internal audit of purchasing practices All are steps in make/buy analysis except reviewing internal purchasing practices.

In its promotional material for bed linens, Scarsdale Corporation states that all sheets and pillowcases sold to customers should have and actually do have a thread count of exactly 600 threads per inch. Scarsdale is apparently emphasizing:

Conformance quality.

3PLs:

Consolidate storage and shipments across customers.

Zanda Corporation's CEO watched in amazement and frustration as she watched workers handling and loading individual products onto trucks. She felt handling the individual items was very cumbersome. She should investigate the possibility of: Using RFID Containerization AS/RS None of these options are correct

Containerization

Zanda Corporation's CEO watched in amazement and frustration as workers handled and loaded individual products onto trucks. She felt handling the individual items was very cumbersome. She should investigate the possibility of:

Containerization.

Which process structure is most likely used by a company that makes plastic soda bottles?

Continuous Process

Jones Company has a make-to-order orientation. It most likely does NOT use:

Continuous Process Structure

Richard's Rollerskate Company relies on Simon's Trucking to transport all of their products. Simon's Trucking is the only shipping company for RRC, and all products are shipped in trucks that look like giant roller skates with Richard's Rollerskate branding on the trucks. Richard's Rollerskate is likely using which carrier type?

Contract carrier

Which of the following statements is true regarding corporate culture?

Corporate culture is learned.

Stockouts are very costly for a medical device company. The logistics manager has decided to hold more inventory to reduce stockouts. This is an example of:

Cost-to-service trade-offs.

One function of warehouses or distribution centers is "consolidation". This means that the warehouse/distribution center is used to: Group small inbound loads into larger outbound loads Collect products/packaging materials that are sent back for disassembly, reclamation, or disposal Hold inventory to buffer the system from variability in either in supply or demand All of the options are correct

Group small inbound loads into larger outbound loads

One function of warehouses or distribution centers is "consolidation." This means that the warehouse/distribution center is used to:

Group small inbound loads into larger outbound loads.

John Jones, senior VP for Zanda Corp., is looking at three alternative aggregate production plans for the next six-month period. At his company, demand varies by month with substantial month-to-month differences. The three alternatives are a "pure level plan," which keeps an absolutely constant workforce, a "pure chase plan" relying on hiring and layoffs, and a hybrid plan. He is most likely to find that the pure level plan:

Has lower inventory carrying cost than the level plan and lower hiring / layoff cost than the chase plan

S&OP has both "hard" and "soft" benefits. Which of the following is not considered a "soft" benefit of S&OP?

Higher sales and enhanced quality

A company's supplier for a critical part was selected based on the lowest purchase price. Six months later, the company's warranty costs began to rise because of defective parts. The company has experienced:

Higher total costs of ownership.

A company's supplier for a critical part was selected based on the lowest purchase price. Six months later, the company's warranty costs began to rises because of defective parts. The company has experienced:

Higher total costs of ownership.

If a company strongly prefers that its aggregate output plan be closer to a level plan than a chase plan, this implies that it is concerned about minimizing:

Hiring and layoff costs

If a company strongly prefers that its aggregate output plan be closer to a level plan than a chase plan, this implies that it is concerned about minimizing:

Hiring and layoff costs.

All of following can increase supply chain risk except: Using a supplier that is having financial difficulty. Holding higher levels of inventory of critical materials. Using suppliers in Thailand. Single sourcing.

Holding higher levels of inventory of critical materials. Holding higher inventory levels is one way to mitigate supply chain risk.

Which of the following is (are) advantage(s) of full partnerships with suppliers, as compared to traditional adversarial relationships? I. They help reduce uncertainties for both buyer and supplier. II. The full partnership is easier to establish and maintain than other relationships, so more suppliers can be dealt with .III. The full partnership assures the lowest material price.

I only

Which of the following is (are) advantages of full partnerships with suppliers, as compared to traditional adversarial relationships? I. They help reduce uncertainties for both buyer and supplier. II. The full partnership is easier to establish and maintain than other relationships, so more suppliers can be dealt with. III. The full partnership assures the lowest material price.

I only.

Currently, our system is characterized by a lack of fit between what the customer wants and what the SCM system is best able to provide. As a result, which of the following would apply? I. The only way to restore fit is for the firm to change the market segment it is going after. II. Unless something is done, our firm will not be the most effective and efficient long-term supplier. III. Any firm can successfully counter such inconsistencies through the use of patents and aggressive advertising.

II only

Which of the following environmental initiatives/programs deals most specifically with the firm's environmental management system? The Kyoto Protocol Responsible Care ISO 14000 LEED certification

ISO

Which of the following environmental initiatives/programs deals most specifically with the firm's environmental management system? ISO 14000 LEED certification The Kyoto Protocol Responsible Care

ISO 14000 Of all the standards and programs summarized in Table 16-1, ISO 14000 is the only one specifically targeted toward the firm's environmental management system.

Walmart has announced that suppliers must attempt to improve their efforts in environmental management. In this announcement, Walmart could have included a requirement that suppliers meet the standards of: SO 9000. ISO 14000. ISO 9002. ISO 2011.

ISO 14000 is the international standard for environmental management.

What is the first step in conducting value engineering/value analysis?

Identify the functional purposes of a product.

A company requires its suppliers to make its products in at least three different locations around the world. Which supply management goal does this BEST accomplish?

Identifying, assessing, and mitigating supply chain risk.

A company that had been using single sourcing, decided to add a second supplier for one of its key raw materials. Which supply management goal does this BEST accomplish?

Identifying, assessing, and mitigating supply chain risk.

A company requires its suppliers to make its products in at least three different locations around the world. Which supply management goals does this BEST accomplish? Fostering sustainability. Reducing total costs. Identifying, assessing, and mitigating supply chain risk. Accessing technology and innovation.

Identifying, assessing, and mitigating supply chain risk. Requiring suppliers to have geographically dispersed operations reduces supply chain risk.

Process thinking causes managers to address critical process elements, including:

If you don't like the outcome, change the process

Process thinking causes managers to address critical process elements, including:

If you don't like the outcome, change the process.

Which of the following might a company try to do to reduce the total amount fo safety stock it holds?

Implement and use ABC analysis of inventory items, attempt to reduce variation in supplier lead times, implemented demand management approaches

Which of the following might a company try to do to reduce the total amount of safety stock it holds?

Implement and use of ABC analysis of inventory items, attempt to reduce variation in supplier lead times, implement demand management approaches

All of the following can improve order processing EXCEPT:

Increasing the number of orders entered manually.

Overall, it is clear that well-managed innovation projects:

Increasingly involve input from multiple functions in a company.

An automated process can be compared to a more labor-intensive process using

Indifference Analysis

Most processes involve two basic types of flows:

Information flows and material flows.

Most processes involve two basic types of flows:

Information flows and physical flows

Most processes involve two basic types of flows: Information flows and physical flows Product flows and decision flows Information flows and operator flows Physical flows and documentation flows

Information flows and physical flows

Understanding the product life cycle is important because:

Innovation investments need to be matched to the phase of a product's life

Floor space used to store items to be inspected belongs to which of the following elements of a process?

Inputs/outputs/ flows

Government regulations belong to which of the following elements of a process? Activities Inputs/outputs/feedback Process structure Management policies

Inputs/outputs/feedback

Random Corp. offers logistics services such as storage, transportation, and information processing to its customers. Random is known in industry as a(n):

Integrated service provider

Random Corp. offers logistics services such as storage, transportation, and information processing to its customers. Random is known in industry as a(n):

Integrated service provider.

Within the hierarchy of planning, the Sales & Operations Planning system occurs at which of the following levels?

Intermediate range

Sales and Operations Planning focuses on what time frame?

Intermediate range planning

If a make-to-stock manufacturing firm with highly seasonal demand follows a level production strategy, which of the following is likely to be true?

Inventory will fluctuate significantly during the year.

Packaging paper towels with six rolls in a package:

Is an example of unitization.

Value stream mapping: Is a simplified version of process mapping Uses the same terminology and symbols used in typical process mapping Is another name for a future state map Correct Answer Is more comprehensive than typical process mapping

Is more comprehensive than typical process mapping

Which of the following statements most accurately represents the challenge of being environmentally sustainable? The choices facing the firm are clear, and the obvious candidates for improvement can be easily identified. It is not easy to be environmentally responsible since the choices facing management involve trade-offs. This is a decision that firms must move on since the demand for environmental responsibility is driven by forces that cannot be ignored—governmental regulations, decreasing resource levels, and changing customer demands. Being environmentally responsible is simply a matter of complying with governmental regulations.

It is not easy to be environmentally responsible since the choices facing management involve trade-offs.

Which of the following statements most accurately represents the challenge of being environmentally sustainable? Being environmentally responsible is simply a matter of complying with governmental regulations. The choices facing the firm are clear, and the obvious candidates for improvement can be easily identified. It is not easy to be environmentally responsible since the choices facing management involve trade-offs. This is a decision that firms must move on since the demand for environmental responsibility is driven by forces that cannot be ignored—governmental regulations, decreasing resource levels, and changing customer demands.

It is not easy to be environmentally responsible since the choices facing management involve trade-offs. As illustrated in the discussion presented in the section, it is not easy to be environmentally responsible since there are significant and conflicting trade-offs that must be addressed—trade-offs where there is no obvious "right" answer.

Which is false with respect to a process structure?

It is the same across an entire supply chain

Which is false with respect to a process structure? A. It is the same across an entire supply chain B. Determines how inputs, activities, flows, and outputs are organized C. Different structures have different capabilities D. The selection of a process structure should consider competitive priorities

It is the same across an entire supply chain

Which of the following is NOT true with respect to an RFQ or RFP? It describes the purchase requirements. It is the first step in competitive bidding. It is often communicated to suppliers electronically. It is used to determine weights in supplier scorecards. Weights in supplier scorecards are determined based on the importance to the organization. An RFQ or RFP is used to communicate the purchase needs to suppliers usually electronically.

It is used to determine weights in supplier scorecards. Weights in supplier scorecards are determined based on the importance to the organization. An RFQ or RFP is used to communicate the purchase needs to suppliers usually electronically.

A well-designed value proposition possesses four characteristics. Which of the following is NOT one of those characteristics?

It offers the highest product quality in each quality dimension.

When calculating a reorder point (ROP), which of the following factors WOULD NOT affect the calculation

Item's EOQ

When calculating a reorder point (ROP), which of the following factors WOULD NOT affect the calculation?

Items EOQ

Lean systems thinking in operations management can extend to sustainability in what way? Its focus on improving product quality. Its focus on delivering products to customers faster. Its focus on reducing waste of all types. Its focus on product innovation.

Its focus on reducing waste of all types.

Lean systems thinking in operations management can extend to sustainability in what way? its focus on delivering products to customers faster . Its focus on reducing waste of all types. Its focus on improving product quality. Its focus on product innovation.

Its focus on reducing waste of all types. Lean systems could improve quality and result in faster delivery, but the focus is on waste reduction, which is important to sustainability.

Three processes have the following costs: Process A has a fixed cost of $2,000 and variable cost of $3.00/unit. Process B fixed cost is $4,000 and variable cost is $2.60/unit. Process C fixed cost is $8,000 and variable cost is $2.40/unit. If the projected total demand is for 6,000 units, which process should be used?

Job Shop

Which of the following process structures can produce the widest variety of products?

Job Shop

Which of the following process structures can produce the widest variety of products? A. Job shop B. Repetitive process C. Continuous flow D. Batch operation

Job Shop

Which of the following process structures can produce the widest variety of products?

Job shop

Zanda Corp. and Jones Corp. are identical in every way (products produced, costs, demand, etc.) except for one. Zanda uses a level production plan while Jones prefers a chase production plan. Which of the following is most likely to be true?

Jones will have lower inventory carrying costs. Jones will have higher investment in plant and equipment. Jones will have higher hiring and firing costs.

A short-term, highly focused effort for improving a process is known as a: Jidoka Event Kaizen Event Chaebol Event Poka-yoke Event

Kaizen Event

Which of the following developments deals with emission trading and clean development mechanisms?

Kyoto Protocol

Which of the following environmental initiatives/programs deals most specifically with building design and construction? The Kyoto Protocol Responsible Care ISO 14000 LEED certification

LEED

Which of the following environmental initiatives/programs deals most specifically with building design and construction? The Kyoto Protocol Responsible Care LEED certification ISO 14000

LEED certification The answer to this question can be found in Table 16-1.

Benefits of single sourcing include all of the following EXCEPT:

Less supply risk.

Benefits of single sourcing include all of the following EXCEPT: Quantity discounts. More consistent quality. Better supplier relationships. Less supply risk.

Less supply risk. Single sourcing increases supply risk.

If the planning horizon is 12 weeks, the cumulative lead time must be:

Less than or equal to 12 weeks.

Compared to a functional layout, cellular layouts have

Less work-in-process inventory

Compared to a functional layout, cellular layouts have:

Less work-in-process inventory

Which type of aggregate production plan is likely to have the LEAST negative impact on the local community and the workforce?

Level plan

Which of the following approaches provides management with an appropriate tool for assessing the full impact from "seed to sewer" (from creation to disposal)? Life cycle analysis Total cost of ownership Trade-off analysis Cost of quality analysis

Life cycle

Which of the following approaches provides management with an appropriate tool for assessing the full impact from "seed to sewer" (from creation to disposal)? Life cycle analysis Trade-off analysis Total cost of ownership Cost of quality analysis

Life cycle analysis Of these four options, only life cycle analysis deals with all of the stages of a product's life.

A bill of materials file contains:

Lists of materials required to produce end item products.

The available capacity of a worker, machine, work center, or facility during a specific period of time is compared against the capacity needed for a specific work schedule in a:

Load profile.

Internal operations managers work with what function to coordinate inbound and outbound flows of materials and information?

Logistics

Logistics management is important to an organization because:

Logistics affects customer service.

Which of the following is NOT a benefit of outsourcing?

Lower supply management costs

Which of the following is NOT a benefit of outsourcing? Greater flexibility to change product specifications Better access to market information Lower supply management costs Less capital needed for investment

Lower supply management costs When outsourcing, a company has to spend more time and effort managing a supplier, so supply management costs are likely to increase rather than to decrease.

Sequential product development projects are likely to have ____________ development costs and ____________ development times than concurrent engineering product development projects.

Lower, longer

The quantities of end items to be completed in each time period into the future is shown in a(n):

MPS

The quantities of end items to be completed in each time period into the future is shown in a(n):

MPS.

Which of the following identifies the ultimate goal of process mapping? Reduce steps in the process. Reduce costs created by the process. Make the desired outcome inevitable Facilitate the identification and reduction of system complexity

Make the desired outcome inevitable

For which market orientation is accurately forecasting the amount of finished goods needed likely to be the most important for a firm's financial performance

Make to Stock

You are given the following data for taillight assemblies: Picture Management has decided to carry out a Pareto analysis focusing on problem areas. Which of the following conclusions does the Pareto analysis support?

Management should focus attention on "inability to meet heat specs" and "scratched lens."

The Williams Co. combines all of its orders going to Tokyo Japan during the month into a single shipment. This is an example of:

Market area consolidation.

Zanda Corp. wants to have a process structure that has cost advantages similar to continuous or repetitive processes, but wants to produce greater variety than those processes normally allow. Zanda should consider:

Mass Customization

Using technology, such as self-checkouts in grocery stores, to enable customers to complete the service delivery and transaction themselves is most commonly referred to as:

Mass Services

Which process structure is most likely used by a company that makes plastic soda bottles?

Mass custimization

Zanda Corp. wants to have a process structure that has cost advantages similar to continuous or repetitive processes, but wants to produce greater variety than those processes normally allow. Zanda should consider:

Mass customization

Using technology, such as self-checkouts in grocery stores, to enable customers to complete the service delivery and transaction themselves is most commonly used for:

Mass services

The primary purpose of an MRP system is to:

Match supply and demand.

The stage of the product life cycle in which process innovation becomes an important way to increase efficiencies is:

Maturity

Joe Jones, plan manager at Waco Industries, told a friend that if it was necessary, his plant could produce 1,000 items a day if all conditions were just right. Joe is describing his plant's:

Maximum Capacity

Joe Jones, plant manager at Waco Industries, told a friend that if it was necessary, his plant could produce 1,000 items a day if all conditions were just right. Joe is describing his plant's:

Maximum Capacity

Joe Jones, plant manager at Waco Industries, told a friend that if it was necessary, his plant could produce 1,000 items a day if all conditions were just right. Joe is describing his plant's:

Maximum capacity

Joe Jones, plant manager at Waco Industries, told a friend that if it was necessary, his plant could produce 1,000 items a day if all conditions were just right. Joe is describing his plant's: Effective capacity Yield rate Utilization Maximum capacity

Maximum capacity

Make/buy/subcontract is typically considered at what time horizon when doing capacity planning? Short-term Medium-term Long-term It is not part of the capacity planning process

Medium-term

The principles of process improvement suggest that an effective, efficient process: Maximizes the number of sequential activities and minimizes the number of parallel ones Eliminates all non-value-added steps Minimizes the number of entry points of a workpiece into the process Minimizes reliance on departments outside of manufacturing

Minimizes the number of entry points of a workpiece into the process

The use of a tablet computer such as an iPad for a restaurant's wine list is an example of:

Mobility

suppliers

Most large corporations require that their ___ have extensive quality management systems such as ISO 9000 and statistical process control

How does aggregate planning for services differ from aggregate production planning for products?

Most service plans are based primarily on labor requirements

How does aggregate planning for services differ from aggregate production planning for products?

Most service plans are based primarily on labor requirements.

Jones Company has identified an item for which the supply risk is high and the value of the purchase to the firm is low. A recommended sourcing strategy for Jones is to:

Multiple Source

Advanced Manufacturing Company is evaluating two suppliers for a component sourcing. After much internal discussion, AMC's management has determined that the critical factors in choosing suppliers are: quality, delivery, price, and service. Further, management has rated the importance of these factors as 0.4, 0.3, 0.2, and 0.1, respectively. On a scale of 1 to 5, Supplier A is rated at 5, 3, 3, and 3, respectively. Supplier B's ratings are 4, 4, 3, and 4. Which supplier has the better weighted score? Supplier A Supplier B Neither, they both have the same weighted score.

Neither, they both have the same weighted score. Supplier A's score = .4(5) + .3(3) + .2(3) + .1(3) = 3.8.Supplier B's score = .4(4) + .3(4) + .2(3) + .1(4) = 3.8

_____ transform product specifications and new process technology into a new or revised production system.

New process design and development projects

Floor cleaning supplies for an elementary school would be considered to be:

Noncritical.

Floor cleaning supplies for an elementary school would be considered:

Noncritical.

Floor cleaning supplies for an elementary school would be considered to be: Leverage. Noncritical. Strategic. Bottleneck.

Noncritical. The level of spend and risk for floor cleaning supplies for a school are both low. This is a noncritical purchase.

An activity identified by a "?" is typically a(n) Value-adding activity Necessary activity Unknown activity None of these

None of these

A disadvantage of "component standardization" is that it:

None of these are disadvantages: Increases development time. Increases development cost. Increases the difficulty of inventory management.

If a company can eliminate all sources of variance in a process: It will be able to operate at maximum capacity at all times There will be no constraints in the process Continuous improvement will not be necessary None of these are true

None of these are true

If a company can eliminate all sources of variance in a process:

None of these are true.

Which is the most important category (as evaluated in terms of weightings used by the outside assessors) of the Malcolm Baldrige National Quality Award Program?

None of these is the most important for the award.

Commitment to "customer success" is likely the best approach for a firm to use with:

None of these selections.

If a company has some customers who express that they are satisfied with the company's performance, the company can count on the fact that:

None of these selections.

negotiation

Normally, ____ is the preferred method of supplier selection for strategic purchases that require full partnerships.

In determining the number of facility locations, the lowest total cost is typically; Not the lowest cost of either transportation or inventory Difficult to generalize Not related to transportation or inventory Typically the lowest cost of either transportation or inventory

Not the lowest cost of either transportation or inventory

Production planning and scheduling is most likely to be associated with which of the following activities? Operation Inspections Storages Transportation

Operation

Value in a process flow analysis is most often associated with which of the following activities? Operation Inspections Storages Transportation

Operation

Which of the following statements is NOT consistent with core values of TQM?

Opinions are as important as data.

The process of removing products for storage and assembling them into appropriate quantities and assortments to fill customer orders is:

Order picking.

The process of removing products from storage and assembling them into appropriate quantities and assortments to fill customer orders is:

Order picking.

Which of the following best describes organizational culture? Organizational culture is easy to change. Organizational culture is readily identified. Organizational culture is becoming less important in today's environment (due to factors such as downsizing and outsourcing). Organizational culture is critical to long-term success.

Organizational culture is critical to long-term success.

Which of the following best describes organizational culture? Organizational culture is becoming less important in today's environment (due to factors such as downsizing and outsourcing). Organizational culture is easy to change. Organizational culture is critical to long-term success . Organizational culture is readily identified.

Organizational culture is critical to long-term success. As discussed in the section dealing with organizational culture, all of the factors except organizational culture being critical to long-term success are not true.

The planned order releases in week 5 are 10 units. Each unit must be processed for five minutes at the workstation. If the available capacity at the workstation is one hour:

Overtime will be needed.

Over which stage of the product's life cycle does operations management have the greatest amount of direct control? Extraction Usage Packaging/transportation Disposal

Packaging/transportation Of these four areas, operations can directly influence packaging/transportation; the others are under the control of other areas (e.g., extraction is under the control of the suppliers and usage and disposal are under the control of the user).

A process having the same activity being performed by two or more resources simultaneously is said to have what kind of structure?

Parallel

You are involved in the implementation of a quality improvement program. Your immediate goal is to identify what problems you should focus on (ideally you want to focus on those that give you the biggest bang for the buck). Which of the following quality control procedures is the most appropriate?

Pareto analysis

The "triple bottom line" suggests that efforts should be devoted to assessing the impact of activities on: People, planet, profit. Profit, planet, processes. People, profit, processes. Profit, profit, profit.

People, planet, profit.

Two dimensions of product quality are hard to measure objectively and are therefore subject to subjective assessment. One of those subjective dimensions is:

Perceived quality

Which of the following is NOT an input into the MRP?

Planned order release

Which of the following pieces of information is NOT contained in an MRP secondary report?

Planned order releases.

. One of the major challenges with a job shop process is that

Planning and scheduling are difficult

The entire time period covered by the MPS is the:

Planning horizon.

MRP is a process that would be most applicable in which situation?

Planning requirements for bicycle seats in a bicycle production plant

MRP is a process that would be most applicable in which situation?

Planning requirements for bicycle seats in a bicycle production plant.

We are experiencing a problem where a process keeps shifting over time. If we were to use a histogram to collect and display the resulting data, what type of shape would it most likely display?

Plateau-shaped distribution

Jones Company learned that several other shippers in its hometown had customers located in the same markets that it serves. With this information, Jones should investigate the possibility of: Market area consolidation Scheduled delivery consolidation Pooled delivery consolidation All of the options are correct

Pooled delivery consolidation

Jones Company learned that several other shippers in its hometown had customers located in the same markets that it serves. With this information, Jones should investigate the possibility of:

Pooled delivery consolidation.

A company decides to reduce its number of warehouse locations (logistically postpone). This decision would result in ___________ or ___________.

Poorer service to customers or higher transportation cost

The width of a histogram showing a distribution of data indicates the __________ of the process that generated the distribution.

Predictability

Precedence relationship

Presents the order in which tasks must be completed

The process of "stapling yourself to an order" refers to: Pretending you are the workpiece moving through a process Observing a process through multiple iterations to eliminate possible biases from observing "random" events. Stapling documents in your process maps to other applicable documents in order not to confuse the "flow." Imagining a workpiece in a process and following it through a process map you have developed.

Pretending you are the workpiece moving through a process

Xanadu Inc. decided to increase the training received by new employees. The expense of this training is an example of which of the following costs of quality:

Prevention costs

Xanadu Inc. decided to increase the training received by new employees. The expense of this training is an example of which of the following costs of quality?

Prevention costs

Ted wants to hire more inspection personnel to insure that all products shipped are free of defects. What TQM core value would this action violate?

Prevention is better than rework

The types of costs included in a cost of quality analysis include:

Prevention, appraisal, and failure.

Which of the following elements of a process most directly impacts or affects process capabilities?

Process Structure

The Taguchi methods link process design to:

Process capability.

A tool used to quickly communicate proposed process changes effectively is called a: Process summary table Future state map Work process flowchart Process flow diagram

Process summary table

Kaizen Events are most appropriate for which type of operations problem? Customer-based Metrics-based Process-based Product-based

Process-based

Back-office processes

Processes that are not seen by the customer (kitchen, phone calls)

Front-office processes

Processes that have contact with the customer (the dining room where the host with interact)

Zanadu Corp. has dedicated equipment and workers in a regularly occurring sequence of activities. Zanadu has which type of operations layout

Product Layout

Zanadu Corp. has dedicated equipment and workers in a regularly occurring sequence of activities. Zanadu has which type of operations layout?

Product Layout

Zanadu Corp. has dedicated equipment and workers in a regularly occurring sequence of activities. Zanadu has which type of operations layout?

Product layout

All of the following are considered when determining total landed costs EXCEPT:

Product liability costs.

The _____________ shows process structures based on output volume and product variety

Product process matrix

Competitive bidding should be used when:

Product specifications are known and clear.

Which of the following is LEAST likely to be considered when determining total landed costs?

Product warranty costs.

Which of the following categories of causes would be most appropriate for a Marriott (an international hotel chain) or McDonalds (an international fast food chain) or the local hair stylist?

Product, Price, Place, Promotion, People/personnel, Process, Physical evidence

Kaizen Events are most appropriate for which type of operations problem?

Product-Based

The _____________ shows process structures based on output volume and product variety.

Product-Process Layout

Which activity is most likely to be outsourced by an automotive company such as Ford Motor Co.?

Production of tires.

Assemble to order (ATO)

Products that are produced from standard components and modules - component and modules can be assembled in different ways to create end product - repetitive process, mass customization (mix coloring agents with white base paint)

Make to order (MTO)

Products that have similar designs but are customized during production - less lead time than ETO - job shop, batch, cellular process structure (a meal at a elegant restaurant, a haircut)

Why do businesses often NOT pursue the best mix of projects in their project portfolio?

Projects are selected based on criteria that are not always consistent with the business strategy.

Which of the following statements is true of projects?

Projects usually involve greater unknowns and uncertainty than repeated routine operations do.

Decision support system

Provide computing power and data management to make higher-quality decisions faster

Sales and Operational Planning:

Provides a regular opportunity for cross-functional teams to review data from the field and to decide how the company should best respond.

Suppose you were the logistics manager for Way-Off Manufacturing and Marketing, a manufacturer of mosquito repellant. Further suppose that Way-Off markets its products in the entire U. S. However, in each geographic region of the country, Way-Off only needs to warehouse products for two months during each year. Way-off probably would use what type of warehousing? Contract Public Private Common

Public

Suppose you were the logistics manager for Way-Off Manufacturing and Marketing, a manufacturer of mosquito repellant. Further suppose that Way-Off markets its products in the entire United States. However, in each geographic region of the country, Way-Off only needs to warehouse products for two months during each year. Way-Off probably would use what type of warehousing?

Public

All of the following are supply management goals EXCEPT

Purchase at the lowest price.

All of the following are supply management goals EXCEPT: Ensure timely availability of resources. Purchase at the lowest price. Enhance quality. Assess technology and innovation.

Purchase at the lowest price. Only purchasing at the lowest price is not a goal of supply management. Rather than reduce purchase price, the appropriate goal is to reduce total cost of ownership.

A legally binding document that signals to a supplier that goods and services are needed is a(n):

Purchase order.

Which of the following statements is false?

Quality dimensions are easy to measure for tangible goods, while they are difficult to measure for services.

Stu Johnson, CEO of Johnson, Inc., attended a seminar on Total Quality Management. Which of the following is NOT something he is likely to have learned at that seminar?

Quality is primarily the responsibility of operations management.

Fundamental areas of management in supply chain operations management include:

Quality, Inventories, and Processes.

From an operations perspective, companies should strive to:

Realize different customer management approaches may be appropriate for different customers.

In ____________, inbound shipments are received and unloaded from the transportation vehicles.

Receiving and unloading

Economic deregulation of transportation can:

Reduce service innovation.

Local sourcing: Reduces delivery costs. Is called "nearshoring". Is used when cost is the primary objective. Takes advantage of lower trade barriers from trade agreements.

Reduces delivery costs. Because of proximity, local sourcing reduces delivery costs.

If a tangible good or service performs the promised function dependably, we say that it has high:

Reliability

Companies with a make-to-stock orientation are most likely to use which of the following process structures? A. Mass customization B. Job shop C. Repetitive process D. They are likely to use any one of these process structures.

Repetitive Process

Which process type is not correctly matched with its descriptive characteristics

Repetitive Process-customized products

Companies with a make-to-stock orientation are most likely to use which of the following process structures

Repetitive Processes

Companies with a make to stock orientation are most likely to use which of the following process structures?

Repetitive process

Which process type is not correctly matched with its descriptive characteristic? A. Project-high complexity B. Repetitive process-customized products C. Job shop-high flexibility D. Continuous flow-low-skilled workers

Repetitive process-customized products

Which of the following is NOT a basic activity type in processes? Delay Operation Reporting Storage

Reporting

Which of the following environmental initiatives/programs is an example of an industry-specific program aimed at improving environmental performance? The Kyoto Protocol Responsible Care ISO 14000 LEED certification

Responsible

Which of the following environmental initiatives/programs is an example of an industry-specific program aimed at improving environmental performance? ISO 14000 Responsible Care The Kyoto Protocol LEED certification

Responsible Care The answer to this question can be found in Table 16-1.

Which of the following is an example of economic regulation?

Restrictions on the type of logistics services a provider can offer.

All of the following are safety and security regulations EXCEPT:

Restrictions on who can form a logistics company.

You observe three different people doing the same task using three different approaches (with differing levels of quality, output, and lead time). Which of the following disposition options is most appropriate for this situation? Keep Rethink Combine Eliminate

Rethink

Returned products are generally handled at a warehouse that provides:

Reverse logistics support.

The process of replanning each period for a given number of periods into the future is referred to as which of the following terms?

Rolling planning horizon

Each month the sales and operations team at Johnson Company meets to develop plans for each of the next six months. This process is known as:

Rolling planning horizons.

Generally speaking, the sales function and operations function differ in objectives. Which of the following is NOT one of those differences?

Sales prefers detailed forecasts for setting bonuses; operations prefers aggregate forecasts.

Suppose that you are a process manager in a manufacturing plant, and you suspect that there is a relationship between the room temperature of the plant and the weights of the plastic caps produced by an injection molding process. What quality improvement tool would you use to investigate this claim?

Scatter diagram

Jones Company has decided to limit deliveries to selected areas to specific days of the week to obtain transportation savings. This is an example of: Market consolidation Transportation consolidation Poor planning Pooled delivery consolidation Scheduled delivery consolidation

Schedule delivery consolidation

Jones Company has decided to limit deliveries to selected areas to specific days of the week to obtain transportation savings. This is an example

Scheduled delivery consolidation.

All of the following represent external failure cost EXCEPT the cost of:

Scrapping defective raw material.

Stockpiling stores inventories in warehouses to protect against

Seasonality in both supply and demand

Which of the following is NOT a role of logistics management?

Selecting raw material suppliers

Which of the following is NOT an activity of integrated logistics management?

Selecting raw material suppliers.

Which of the following is NOT a role of logistics management?

Selecting suppliers for parts and components.

approach that analyzes the interface between customers and service processes is called

Service Blue Printing

An approach that analyzes the interface between customers and service processes is called

Service Blueprinting

Choose the service type that is correctly matched with one of its key characteristics

Service Factory--facilities and equipment represent a large proportion of total costs

Choose the service type that is correctly matched with one of its key characteristics

Service factory- facilities and equipment represent a large proportion of total costs

Choose the service type that is correctly matched with one of its key characteristics.

Service factory-facilities and equipment represent a large proportion of total costs.

Choose the service type that is correctly matched with one of its key characteristics. A. Service factory-facilities and equipment represent a large proportion of total costs. B. Service shop-there is a low degree of customer interaction. C. Mass service-transactions are very varied from customer to customer. D. Professional service-transactions tend to be fairly standardized.

Service factory-facilities and equipment represent a large proportion of total costs.

What is the major aggregate planning difference between service and manufacturing firms?

Service firms cannot hold inventory of their product.

Implementation of a Six Sigma program typically involves:

Setting up training at various levels including green belt and black belt training.

Which of the following is an example of an activity that is "necessary but not value-adding"? Scheduling Processing Inspections Setups

Setups

Planning for which of the following products would see the LEAST benefit from the use of an MRP system?

Shampoo

________ involves verifying that the assembled orders are correct and the actual loading of the transportation vehicles occurs.

Shipping

Squeaky Klean, a small consumer products company, purchases custom-designed plastic bottles for its cleaning products that are made at a single plant in Cleveland, Ohio. To make these bottles, specialized tooling must be purchased at a high cost and installed at a supplier's plant. Its sourcing strategy for bottles should be to:

Single source.

Hugging in a control chart usually indicates:

Something is occurring that is masking natural variation.

The identification, evaluation, selection and management of suppliers is called:

Sourcing.

The identification, evaluation, selection and management of suppliers is called: Sourcing. Supply management. Supply chain management. Spend analysis.

Sourcing. Sourcing is the identification, evaluation, selection of and management of suppliers.

With respect to the following operating service characteristics of the modes of transportation, which combination is CORRECTLY specified?

Speed: air is best; pipeline is worst.

Aspen Corporation has discovered that it uses 10 different suppliers for a particular type of item, has bought 20 different models of that item, and total expenditures last year for the item were $100,000. Aspen most likely is conducting:

Spend analysis

The process of understanding how a firm is spending its money and with which suppliers is called:

Spend analysis.

Aspen Corporation has discovered that it uses 10 different suppliers for a particular type of item, has bought 20 different models of that item, and total expenditures last year for the item were $100,000. Aspen most likely is conducting: Supplier audits. Spend analysis. Supplier certification. Make or buy analysis.

Spend analysis. Spend analysis refers to analyzing the dollars spent with which suppliers.

leverage purchases

Standardize purchases across the company, use competition to select suppliers, and consolidate purchases with one or a few suppliers to get discounts.

Which approach is likely to be used for leverage purchases?

Standardizing purchases

Which approach is likely to be used for leverage purchases? Purchasing cards Vendor-managed inventory Using executive champions Standardizing purchases

Standardizing purchases Standardizing purchases across the company increases the purchase volume needed for leverage purchases. Purchasing cards and VMI are used for noncritical purchases, and executive champions are used for strategic purchases.

The 'triangle' symbol indicates what type of activity in a process? Operation Transportation Delay Storage

Storage

core competencies

Strategically, organizations must focus on their current or future ____ ___ .

A maager is currently unhappy with the capabilities of a particular process and wants to improve it capabilities. To accomplish this the manager needs to focus on the process:

Structure

A manager is currently unhappy with the capabilities of a particular process and wants to improve its capabilities. To accomplish this, the manager needs to focus on the process: Inputs, outputs, and flows Activities Management metrics Structure

Structure

A company has decided that it no longer needs to extensively count and inspect the products it receives from a particular supplier. This suggests that the purchasing company has begun:

Supplier certification.

A comprehensive system, facilitated by software, that manages the firm's interactions with its supply base is:

Supplier relationship management.

Williams Inc. has acquired software to help manage interactions with its supply base. This suggests that Williams Inc. is involved in:

Supplier relationship management.

In its own country, a former supplier to your company recently began selling its own product that is identical to a product that was developed by your company. This is an example of:

Supply chain risk.

In its own country, a former supplier to your company recently began selling its own product that is identical to a product that was developed by your company. This is an example of:

Supply risk

In its own country, a former supplier to your company recently began selling its own product that is identical to a product that was developed by your company. This is an example of: Supply risk. Sustainability. Insourcing. Strategic sourcing.

Supply risk. Theft of intellectual property is one type of supply risk.

If you were developing a service blueprint, how would you classify "delivery of gasoline to a service station

Support Processes

If you were developing a service blueprint, how would you classify "delivery of gasoline to a service station"?

Support processes

Which of the following terms best describes the impact of sustainability initiatives on the business model? Sustainability affects primarily the capabilities offered by the supply chain. Sustainability helps the firm to change its value proposition. Sustainability impacts primarily the customers that we serve. Sustainability affects the entire business model.

Sustainability affects the entire business model

Which of the following terms best describes the impact of sustainability initiatives on the business model? Sustainability affects primarily the capabilities offered by the supply chain. Sustainability helps the firm to change its value proposition. Sustainability affects the entire business model. Sustainability impacts primarily the customers that we serve.

Sustainability affects the entire business model.

Which of the following statements best describes the role of sustainability measurement and reporting in many of today's firms? Sustainability reporting is becoming more important, with its structure being determined by governmental regulations and requirements. Sustainability reporting focuses on those issues where the reporting firm has the greatest degree of control. Sustainability reporting is becoming more important with more metrics being provided by different organizations. Sustainability reporting, while important, is plagued by problems involving its development and deployment and with concerns over appropriate standards.

Sustainability reporting is becoming more important with more metrics being provided by different organizations

Which of the following statements best describes the role of sustainability measurement and reporting in many of today's firms? Sustainability reporting focuses on those issues where the reporting firm has the greatest degree of control. Sustainability reporting is becoming more important, with its structure being determined by governmental regulations and requirements. Sustainability reporting, while important, is plagued by problems involving its development and deployment and with concerns over appropriate standards. correct Sustainability reporting is becoming more important with more metrics being provided by different organizations.

Sustainability reporting is becoming more important with more metrics being provided by different organizations. Of these various statements, all are essentially false except for the right answer.

Which of the following mapping techniques identifies and integrates functional responsibilities into the process mapping process? Process flow diagram Value stream mapping Service blueprinting Swim lanes

Swim lanes

A primary reason for using global suppliers is to: Improve sustainability. Reduce transportation costs. Take advantage of free trade agreements such as NAFTA. Work with the same suppliers in many different regions of the world.

Take advantage of free trade agreements such as NAFTA. When expanding globally, many companies want their suppliers to be able to expand with them.

In line-balancing, takt time is the time

That is the maximum allowable at each workstation based on customer demand

A process can be said to be capable when:

The Cpk is greater than 1.5.

sourcing strategy

The ___ ___ determines which suppliers to use for which purchases.

Why is product/process innovation an important driver of firm performance?

The ability to imitate as well as innovate

An organizational culture is shaped by:

The actions taken by leadership

A company has decided to outsource the production of key components for its main product to suppliers. What effect will this have on the product's bill of materials?

The bill of materials will be flatter.

Suppose you are shipping 1,000 pounds of product to a customer location that is 500 miles away from you. The customer calls you and increases the order size to 10,000 pounds. In this case:

The cost per pound of transportation for the order will decrease.

As the shipping distance increases

The cost per unit of distance decreases.

As the shipping distance increases:

The cost per unit of distance decreases.

Qualitative factors that should be assessed when making an insourcing/outsourcing decision include all of the following EXCEPT:

The costs incurred at the start of the contract

If a make-to-stock manufacturing firm with highly seasonal demand follows a chase demand strategy, which of the following is likely to be true?

The firm likely will have higher capital investment than if it followed a level plan

If a make-to-stock manufacturing firm with highly seasonal demand follows a chase demand strategy, which of the following is likely to be true?

The firm likely will have higher capital investment than if it followed a level plan.

Fixed costs per order

The firm will incur costs each time it places a new order.

Engineer to order (ETO), Make to order (MTO), Assemble to order (ATO), Make to stock (MTS)

The four types of process structure and market orientation

Fixed position layout

The layout used when the product cannot be moved during production - project (homes, building, ship)

A bottleneck activity in a process is generally the activity with:

The least capacity

A bottleneck activity in a process is generally the activity with: The smallest number of resources devoted to it The least capacity No delays in front of it The highest cost

The least capacity

A bottleneck activity in a process is generally the activity with:

The least capacity.

When you need to identify a supplier for a new purchase, the FIRST place that you should look is:

The list of your company's current suppliers.

Takt time

The maximum allowable cycle time at each workstation (Available production time per day/output needed per day)

Zanda Corp. has always operated within one country. It is contemplating opening production facilities in another country. Zanda should realize: Production processes used domestically should work well in a different country. The means of motivating the workforce domestically may not work well in a different country. The technology used domestically is often the best to use in a different country. The same business model that is used domestically should apply to the foreign country.

The means of motivating the workforce domestically may not work well in a different country.

Zanda Corp. has always operated within one country. It is contemplating opening production facilities in another country. Zanda should realize: The means of motivating the workforce domestically may not work well in a different country. Production processes used domestically should work well in a different country. The technology used domestically is often the best to use in a different country. The same business model that is used domestically should apply to the foreign country.

The means of motivating the workforce domestically may not work well in a different country. Because of cultural differences, including skills, ways of interacting, what constitutes "success," may be different in another culture.

To an operations manager, the "key customer" is:

The person who has the greatest impact on design, sales, and growth opportunities for the product.

Customer contact

The presence of the customer in a process

Outsourcing is least likely when: The process in highly innovative. Products are in the mature phase of the lifecycle. The production processes are standard. The activity Is not strategically important.

The process in highly innovative. When a product is highly innovative it is more likely to be insourced to protect intellectual property.

You have been given the following data: Specification information: Anything between 10 and 20 units per order is acceptable. Process Mean: 13 Upper limit of the process: 16 Lower limit of the process: 10 Using this information, which of the following conclusions is most appropriate?

The process is barely capable but not centered.

A firm has an existing process that it would like to use for producing a part it will sell to an industrial customer. The customer requires that defective (out of specification) parts be kept to 3,000 parts per million (three-tenths of 1 percent) or less. A process capability study done on the process revealed that it has a process capability index (Cpk) of 1.00 for this part. What should the firm conclude from the study?

The process is capable, but just barely, and will require careful monitoring in order to meet customer requirements.

A supplier of materials used in manufactured goods is a member of:

The product supply chain

Cellular manufacturing

The production of products with similar process characteristics on small assembly lines called cells (Job shop, batch)

An MPS shows:

The quantities of end items to be completed in each time period.

"Product quality" is best explained as:

The result of both design quality and conformance quality

."Product quality" is best explained as:

The result of both design quality and conformance quality

"Concurrent engineering" is best described as:

The simultaneous design and development of all the processes and information needed to manufacture a product, to sell it, to distribute it, and to service it.

Which of the following best describes the concept of total landed cost?

The sum of all product-related and logistics-related costs.

With respect to transportation regulation today in the United States:

There is more safety/social regulation than ever before

With respect to transportation regulation today in the United States: There is very little safety/social regulation There is no economic regulation of transportation There is more safety/social regulation than ever before There is more economic regulation than ever before

There is more safety/social regulation than ever before

With respect to transportation regulation today in the United States:

There is more safety/social regulation than ever before.

Variable costs

These are costs associated with each unit produced, including labor, materials, asset depreciation, energy, or the purchase price.

Fixed costs per contract

These are the one-time costs incurred by the buying firm at the start of the contract or when beginning to make a product.

Why are processes that are visible to the customer considered to be critical? They often exhibit the greatest variance (thus affecting customer waiting times). They are often bottlenecks (thus causing customer delays). They are often shared processes. They affect the customer perceptions of value.

They affect the customer perceptions of value.

High-quality innovators typically gain which advantages?

They satisfy customers more effectively, building strong brand image and customer loyalty.

Jones Manufacturing sells a part to Lear Corporation. Lear puts this part into a radio, which Lear then sells to Ford. From Ford's point of view, Jones Manufacturing is a(n) __________ supplier.

Tier 2

purchase requisition

To communicate details such as what, when, and how many to the supply management department, people use an internal company document referred to as a ___ ___.

What is the primary objective of line balancing?

To match output rates with actual demand

Which of the following phrases BEST defines the concept of sustainability, as set out by the United Nations Brundtland Commission: To reduce, minimize, and ideally eliminate pollution, thus preserving the planet and its resources for the benefit of not only today's generation but also the future. To meet the needs of the present without compromising the ability of future generations to meet their needs. To ensure that business and the community work together to preserve limited resources while also ensuring that business is appropriately rewarded for its efforts. To foster an environment that satisfies the requirements of the triple bottom line—namely people, profit, and the planet—the keys to our future survival.

To meet the needs of the present without compromising the ability of future generations to meet their needs

Which of the following phrases BEST defines the concept of sustainability, as set out by the United Nations Brundtland Commission: To meet the needs of the present without compromising the ability of future generations to meet their needs. To ensure that business and the community work together to preserve limited resources while also ensuring that business is appropriately rewarded for its efforts. To foster an environment that satisfies the requirements of the triple bottom line—namely people, profit, and the planet—the keys to our future survival. To reduce, minimize, and ideally eliminate pollution, thus preserving the planet and its resources for the benefit of not only today's generation but also the future. The answer is found in the first paragraph under the heading, and it focuses on the notion that sustainability must deal with needs (not pollution alone) and that these needs must

To meet the needs of the present without compromising the ability of future generations to meet their needs. The answer is found in the first paragraph under the heading, and it focuses on the notion that sustainability must deal with needs (not pollution alone) and that these needs must be satisfied both today and in the future.

The objective of the "control" step in the DMAIC process is:

To prevent an improved process from becoming highly variable again.

The 'arrow' symbol indicates what type of activity in a process? Operation Transportation Delay Storage

Transportation

Products are received, sorted, sequenced, and selected into loads at a:

Transshipment point.

Engineer to order (ETO)

Unique, customized product - require long lead time - project, job shop process structure (custom built house, a customized employee training program)

BMW has introduced a program of covered scheduled maintenance for its cars. That is, when you bring in your vehicle for maintenance at a BMW dealership, BMW will cover the costs of everything (except for worn-out tires). This practice specifically targets which stage of the product life cycle? Extraction Packaging/transportation Usage Disposal

Usage

BMW has introduced a program of covered scheduled maintenance for its cars. That is, when you bring in your vehicle for maintenance at a BMW dealership, BMW will cover the costs of everything (except for worn-out tires). This practice specifically targets which stage of the product life cycle? Usage Disposal Packaging/transportation Extraction

Usage What BMW is doing here is influencing how the customer uses (and treats) the product, thus improving the trade-in values and reducing the total costs of ownership. Consequently, the correct answer is Usage.

bottleneck purchases

Use at least two suppliers to assure supply, develop new suppliers, and explore using different materials.

Jones Company has identified an item for which the supply risk is high and the value of the purchase to the firm is low. A recommended sourcing strategy for Jones is to:

Use multiple sourcing

Strategic purchases

Use one or two suppliers and build partnerships with them to foster collaboration and innovation.

noncritical items

Use vendor-managed inventory and allow users to make their own purchases using online catalogs or corporate credit cards (called purchasing cards) to lower the transaction costs of purchasing.

Line balancing

Used to assign tasks so that idle time and the number of workstations are minimized

Mass customization

Uses advanced technologies to customize products quickly and at a low cost (Dell computers, able to select range of components and warranty options)

Distribution requirements planning:

Uses logic similar to MRP, beginning at the point closest to the customer.

Adam Smith described his company's process as follows: "Under ideal conditions we can produce 10,000 units a day. However, our normal production is 6,000 units a day. Today, we actually produced 8,000 units." Which of the following is true?

Utilization was 133 percent effective capacity

Adam Smith described his company's process as follows: "Under ideal conditions we can produce 10,000 units a day. However, our normal production is 6,000 units a day. Today, we actually produced 8,000 units." Which of the following is true?

Utilization was 133 percent of capacity

Adam Smith described his company's process as follows: "Under ideal conditions we can produce 10,000 units a day. However, our normal production is 6,000 units a day. Today, we actually produced 8,000 units." Which of the following is true? Utilization is 60 percent of maximum capacity Yield is 133 percent of effective capacity Utilization was 133 percent of effective capacity All of these are true

Utilization was 133 percent of effective capacity

Adam Smith described his company's process as follows: "Under ideal conditions we can produce 10,000 units a day. However, our normal production is 6,000 units a day. Today, we actually produced 8,000 units." Which of the following is true?

Utilization was 133% of effective capacity

Which of the following is NOT considered in the service process matrix

Volume

Which of the following is NOT considered in the service process matrix? A. Customization B. Volume C. Customer interaction D. Labor intensity

Volume

One implication of increased utilization in a process is that: Process cost typically will go up Waiting time typically will go up Variance typically will be reduced All of these

Waiting time typically will go up

You are given the following information. Which of these statements can you support with this information? Maximum Capacity: 480 hours per week Effective Capacity Ratio: 85% Demonstrated Capacity: 380 hours per week over the past two weeks On-time Delivery %: 75% of the jobs are being completed on-time More capacity needs to be added in the short term to improve performance in the system We have to look at the rate at which jobs are being input into the shop Our workforce is not working hard enough We need to further investigate whether the problem is in input or capacity

We need to further investigate whether the problem is in input or capacity

All of the following typically use a "make to stock" market orientation except:

Wedding Cake

Under what conditions would we be required to recalculate the parameters for an X-bar-R chart?

When we change the production process.

"Bounding" a process is necessary because: It determines which activity is the bottleneck If the project is "shelved" during your analysis, someone else may have to pick up where you left off. Bounding determines if a process is a core competency Without bounding, it is difficult to determine the beginning and ending points for the analysis.

Without bounding, it is difficult to determine the beginning and ending points for the analysis.

. Which of following is NOT a drawback to a product layout: A. Lack of flexibility to change product features B. A problem at any workstation can stop the entire production line C. Work-in-process is typically very high D. The potential for employee boredom

Work-in-process is typically very high

Every year around mid-May, the Grand Hotel on Mackinac Island runs a special to coincide with the Lilac Festival. This special offers visitors a three day stay (3 days/2 nights) combined with all meals and a guided tour of the various Lilac displays. Which of the following terms best captures this program?

Yield Management

You are sitting next to a person in business class on a flight from LA to Sydney, Australia. You mention to that person that you got your ticket two months ago for only $12,500. The person responds that she bought her ticket two days ago for $7,800. This sometimes happens because airlines often use an approach called:

Yield Management

You are sitting next to a person in business class on a flight from Los Angeles to Sydney, Australia. You mention to that person that you got your ticket two months ago for only $12,500. The person responds that she bought her ticket two days ago for $7,800. This sometimes happens because airlines often use an approach called:

Yield management.

Arm's-length

___ ___ relationships are appropriate for bottleneck or noncritical items

price

___ is rarely the most important factor in negotiation

Risks

____ are assessed based on their probability of occurrence and their impact on the firm.

"Pass/ fail" and "go/no go" are both examples of: Attribute data Decision data Process flow data Variable data

a

"Quality at the source" suggests quality should be: Built into the product Inspected at multiple points Ensured by the quality control lab An inspector's function

a

All of the following represent external failure cost EXCEPT the cost of: Scrapping defective raw material Repairing items covered by warranty Replacing defective items found by customers Loss of goodwill and reputation

a

An application of the DMAIC process to product design is known as: Design for Six Sigma Design for Quality Quality Function Deployment None of these answers are correct.

a

At the heart of the Six Sigma program is a five-step process called D-M-A-I-C. These letters denote a major activity that must be completed to achieve the objectives of Six Sigma. These activities are: Define, Measure, Analyze, Improve, Control Define, Measure, Analyze, Inquire, and Compare Define, Measure, Analyze, Involve, and Cost Define, Measure Analyze, Improve, and Compare

a

Sample means and ranges were obtained for five samples of 10 units per sample from a production process. Assume the process was considered to be in control during the period these samples were collected. The results are as follows: Sample # Mean Range 1 10.58 .23 2 9.97 .25 3 9.99 .21 4 10.23 .28 5 10.23 .28 The LCL for the three sigma X chart for the process is approximately (for a sample size of 10, A2 = 0.31): 10.12 10.05 10.94 9.44

a

The specifications for the diameter of a molded part are 10 mm ± 0.5 mm. The actual average and standard deviation from 250 parts sampled is 10.1 mm and 0.1 mm, respectively. The process capability as measured by Cp is 1.67, and by Cpk is 1.34. The process analyzed can be characterized as: Capable Unreliable Just barely capable Out of control

a

The width of a histogram showing a distribution of data indicates the __________ of the process that generated the distribution. Predictability Average Bias Central tendency

a

Which of the following tools is most appropriate when we are interested in determining if a process that was generating attribute-type data (pass/fail, good/bad) was under control? p control chart Xbar-R control chart Scatterplot Histogram

a

Which of the following products would be most likely "make to stock"?

a car

Which product is likely to use a fixed-position layout in its production process?

a commercial plane

Which of the following is the best example of a service shop?

a computer repair shop

job shop

a flexible process structure for products that require different inputs and have different flows through the process (e.g. auto repair shops, beauty salons, department stores); variety of products so employees and equipment must be flexible; labor intensive, produced in low volume, job sequencing is challenging, and have high WIP inventories

functional layout

a layout that groups together similar resources; problems occurring at a single workstation does not usually stop production because it offers many different routes for a given job

product layout

a layout where resources are arranged according to a regularly occurring sequence of activities; the flow of products or customers is visible and easy to trace, lack of flexibility and low work variety for employees are drawbacks, and are typically used by repetitive and continuous processes

project

a one-time or infrequently occurring set of activities that create outputs within prespecified time and cost schedules (e.g. building a custom home, designing a video game, or planning a wedding); customized so customer is highly involved and employees and equipment must be flexible

repetitive process

a process in which discrete products flow through the same sequence of activities (e.g. appliances, automobiles, buffet restaurant); high volume for standard products with a limited range of options; employees are not highly skilled but become very efficient

batch process

a process in which goods or services are produced in groups (batched) and not in a continuous stream (e.g. bakeries, automotive parts, cinema); good for basic models with several different options produced in moderate levels of variety and volume

continuous process

a single-flow process used for high-volume, non-discrete, standardized products (e.g. aluminum cans, laundry detergent, gasoline); commodities that are made-to-stock, line stoppages are very costly, follow sequence, run 24/7, and highly-specialized equipment and low-skilled operators are used

front-office/visible contact employee actions

actions of frontline contact employees that occur as part of a face-to-face encounter with customers

support processes

all activities carried out by employees who do not have direct contact with customers, but whose function are crucial to the service (prepare food)

support processes

all activities carried out by employees who do not have direct contact with customers, but whose functions are crucial to the service

customer actions

all of the steps that customers take as a part of the service delivery process

physical evidence

all of the tangibles that customers see or collect during their contact with a company

Online reverse auctions

allow suppliers to competitively bid for a buyer's business in real time

service blueprinting

an approach similar to process mapping that analyzes the interface between customers and service processes (focuses on front-office and back-office actions, support processes, physical evidence, and customer actions)

Which of the following products would be most likely "make to stock"

an automobile

Which of the following products is most likely to require an assemble to order market orientation?

an upholstered sofa

Which of the following products is most likely to require as assemble to order market orientation?

an upholstered sofa

Leverage purchases

are low risk but represent a high level of spend. They typically involve standard goods or services where many possible suppliers are available.

back-office contact employee actions

are non visible interactions with customers (take bags to room)

If the cumulative lead time is 8 weeks, for a project to be feasible the MPS planning horizon must be:

at least 8 weeks

What might a company do to reduce the amount of cycle stock it holds?

attempt to reduce order costs

mass customization

attempts to maintain cost advantages similar to continuous or repetitive processes, but allows a firm to produce greater variety than those processes normally allow

processing technologies

automate material and data processing to provide 24/7 resource availability, faster processing, greater consistency, and lower cost (e.g. industrial robot)

A packaging plant is running a filling process where packages are filled with 24 oz. +/- 0.08 oz. A capability study reveals that the process mean is 24 oz. with a standard deviation of 0.02 oz. What is the capability of the packaging line? 0.67 1.33 0.50 1.50

b

Forty samples of 100 are taken, with the total number of defective units being 150. What is the upper control limit of the three sigma (z = 3) p-chart? 0.0375 0.0945 0.165 0.793

b

If a company spends more on prevention, what would be the expected impact on other costs of quality? Appraisal costs will go up Failure costs will go down Failure costs will go up Other costs will be unaffected

b

If a product includes features that customers care most about, we would say that it has high: Conformance quality Design quality Reliability Aesthetics

b

Implementation of a Six Sigma program typically involves: Applying for the Malcolm Baldrige Award Setting up training at various levels including green belt and black belt training A cost of quality analysis None of these are usually involved in implementing Six Sigma

b

In its promotional material for bed linens, Scarsdale Corporation states that all sheets and pillowcases sold to customers should have and actually do have a thread count of exactly 600 threads per inch. Scarsdale is apparently emphasizing: Design quality Conformance quality Product quality Total quality management

b

Sanford Corp. bought new technological systems to inspect the quality of products as they come off the production line. The expense of operating these systems would be an example of which of the following types of quality-related costs? Internal failure cost Appraisal cost External failure cost Prevention cost

b

Stu Johnson, CEO of Johnson, Inc., attended a seminar on Total Quality Management. Which of the following is NOT something he is likely to have learned at that seminar? Some aspects of quality are difficult to measure because they depend on customers' judgments Quality is primarily the responsibility of operations management Quality results from design as well as operations and delivery Cross-functional teams are important in TQM decision making

b

The Plan-Do-Check-Act cycle is best defined as: A rigorous procedure for strategic planning A method used to guide problem identification and solution A method first proposed by Joseph Juran All of these statements apply to the definition

b

The TQM view of organizational structure states that Employees must support management decisions Correct Answer Management must support employees Customers want quality products Middle management must support and enforce top management decisions

b

The specifications for the diameter of a molded part are 10 mm ± 0.5 mm. The actual average and standard deviation from 250 parts sampled is 10.1 mm and 0.1 mm, respectively. What is the process capability as measured by Cp? 0.84 1.67 0.60 1.00

b

We are experiencing a problem where a process keeps shifting over time. If we were to use a histogram to collect and display the resulting data, what type of shape would it most likely display? Bell (normal) distribution Plateau-shaped distribution Bimodal distribution Skewed distribution

b

What decisions do supply managers make that potentially impact quality? Design of manufacturing and service processes Design of contracts and associated incentives and penalties Design of packaging Design of product prototyping procedures

b

You have been given the following data: Specification information: Anything between 10 and 20 units per order is acceptable. Process Mean: 13 Upper limit of the process: 16 Lower limit of the process: 10 Using this information, which of the following conclusions is most appropriate? The process is not capable since it is not centered. The process is barely capable but not centered The process is capable and centered The process is not capable and centered

b

When small changes generated by a customer produce progressively larger changes at each stage upstream in the supply chain, this is known as:

bullwhip effect

"Zero defects" is a concept of quality management of which thought leader? Deming Juran Crosby Imai

c

If a tangible good or service performs the promised function dependably, we say that it has high: Conformance Durability Reliability. Perceived quality

c

Moderate- to high-risk purchases with low to moderate levels of spend are categorized as: A) Noncritical. B) Leverage. C) Bottleneck. D) Strategic.

c

Sample means and ranges were obtained for five samples of 10 units per sample from a production process. Assume the process was considered to be in control during the period these samples were collected. The results are as follows: Sample # Mean Range 1 10.58 .23 2 9.97 .25 3 9.99 .21 4 10.23 .28 5 10.23 .28 What is the central line of the X chart for this process 10.00 10.05 10.20 9.45

c

Squeaky Klean, a small consumer products company, purchases custom-designed plastic bottles for its cleaning products that are made at a single plant in Cleveland, Ohio. To make these bottles, specialized tooling must be purchased at a high cost and installed at a supplier's plant. Its sourcing strategy for bottles should be to: A) Use three or more suppliers. B) Dual source. C) Single source. D) Insource bottle production

c

The Taguchi methods link process design to: Customer expectations Production availability Process capability All of these

c

The initial focus of the DMAIC process is on: Process variability Cost Critical-to-quality characteristics Problem solving

c

The managers of a production line that fills cereal boxes would like to reach a Six Sigma level of quality in terms of product weight. The desired range of output extends from 10 to 16 ounces. In order to reach Six Sigma quality, what would the maximum standard deviation of output need to be? 1 ounce 2 ounces 0.5 ounces Not enough information is given to answer the question

c

The objective of statistical process control (SPC) is to: Determine whether a batch of raw materials should be accepted or rejected Determine whether a batch of finished goods should be accepted or rejected Detect assignable cause variations versus normal random variations in the process Identify variable versus attribute measures

c

Under what conditions would we be required to recalculate the parameters for an X-bar-R chart? When the first sample mean crosses either the UCL or LCL When there is hugging observed in the data When we change the production process When dealing with a two-shift operation

c

Which of the following categories of causes would be most appropriate for a Marriott (an international hotel chain) or McDonalds (an international fast food chain) or the local hair stylist? People, Machine/Equipment, Methods/Processes, Materials Machine, Method, Material, Manpower/Mindpower, Measurement, Mother Nature Product, Price, Place, Promotion, People/personnel, Process, Physical evidence People, Price, Machine, Materials, Mother Nature

c

Which of the following is NOT a core value of TQM? Prevention is better than inspection Problem solving should occur at the lowest organizational levels possible Quality assurance managers should have the most power in the organization Product quality should be viewed holistically

c

XYZ distribution center receives 1,000 deliveries each day. Arrivals are on time if they are within plus or minus 10 minutes of the targeted arrival. In order to have only 3.4 late or early arrivals per million deliveries, what would the standard deviation of arrival times need to be? 3.33 minutes 0.83 minutes 1.67 minutes 6.67 minutes

c

You are involved in the implementation of a quality improvement program. Your immediate goal is to identify what problems you should focus on (ideally you want to focus on those that give you the biggest bang for the buck). Which of the following quality control procedures is the most appropriate? Fishbone diagrams Process flow charts Pareto analysis Histograms

c

Zanda Corp. has determined that it has too many products returned from customers. Zanda is interested in determining what factors or elements could contribute to the large number of returns. Which of the following quality tools would Zanda use to achieve this objective? Process flowchart Pareto analysis Fishbone diagram Run diagram

c

Which of the following is NOT one of the costs considered in aggregate production planning?

capital equipment cost

Taxes and insurance costs are an example of which of the following costs?

carrying (or holding) costs

product-process matrix

categorizes processes into structures based on output volume and variety

service process matrix

categorizes service processes based upon the degree of customization/customer interaction and labor/capital intensity

Significant changes in an MPS quantity several weeks into the planning horizon can:

cause inconsistencies in the MRP

flexible manufacturing systems (FMS)

combine automated machines, robots, and material handling systems that are all controlled by a single computer

Continual improvement of process is important because:

competitors are improving their processes, customer preferences change over time, technology is constantly changing

total cost of ownership (TCO)

considers all the costs incurred before, during, and after the purchase of a good or service.

Which process structure is most likely used by a company that makes plastic soda bottles?

continuos process

A firm has an existing process that it would like to use for producing a part it will sell to an industrial customer. The customer requires that defective (out of specification) parts be kept to 3,000 parts per million (three-tenths of 1 percent) or less. A process capability study done on the process revealed that it has a process capability index (Cpk) of 1.00 for this part. What should the firm conclude from the study? The process is not capable, but with careful process control, it could be used to meet the customer's requirements The process is highly capable, and when operating in control, a very high percentage of output will meet the customer's requirements The process is not capable, and even with careful control, it will not be able to meet the customer's requirements The process is capable, but just barely, and will require careful monitoring in order to meet customer requirements

d

A primary reason for using global suppliers is to: A) Improve sustainability. B) Reduce transportation costs. C) Take advantage of free trade agreements. D) Work with the same suppliers in many different regions of the world.

d

A process can be said to be capable when: The Cp is greater than 1.5 The Cpk is less than 1.0 The process is perfectly centered within the specification The Cpk is greater than 1.5

d

Benefits of single sourcing include all of the following EXCEPT: A) Quantity discounts. B) More consistent quality. C) Better supplier relationships. D) Less supply risk.

d

Fifteen samples of size 10 are taken from a stable process. The average of the sample means is 32.8, and the average range of the samples is 1.5. What is the upper control limit for the R chart? Use the information contained in the attached chart. Swink 2e - Chap 06S - Q28 (DAF 07-13) 1.777 1.653 2.666 2.480

d

Hugging in a control chart usually indicates: The process is out of control Systematic problems in the process that require attention The process is in control Something is occurring that is masking natural variation

d

Suppose that you are a process manager in a manufacturing plant, and you suspect that there is a relationship between the room temperature of the plant and the weights of the plastic caps produced by an injection molding process. What quality improvement tool would you use to investigate this claim? Fishbone diagram Check list Pareto analysis Scatter diagram

d

The specifications for the diameter of a molded part are 10 mm ± 0.5 mm. The actual average and standard deviation from 250 parts sampled is 10.1 mm and 0.1 mm, respectively. What is the process capability as measured by Cpk? 0.81 1.01 0.33 1.34

d

The types of costs included in a cost of quality analysis include: Production, delivery, and marketing Internal and external Direct and indirect Prevention, appraisal, and failure

d

Why is product quality sometimes poorly defined in a firm? Managers do not know how to measure customer service Holistic measures of quality are difficult to develop Managers in different functional areas tend to emphasize different dimensions of quality Both B and C are correct

d

You are given the following data for taillight assemblies. Which of the following tools does this data represent? Orders 7-9 am 9-11 am 11-1 am 1-3 pm 3-5 pm Sum Insufficent Plating vvvvvvvv v vvvvvv 15 Inability to meet heat specs vvvvvv vvvvvvvvv vvvvvvvvvv vvvvvvvvv vvvvvvvv 42 scratched lens vvvvvvvvvv vv vvvv vvv vvvvvvvvvv v 28 failed leak test vv vv v vvv vv 12 glue on lens vvv vvv v 7 cracked body v vv vv 5 Sum 27 16 18 20 28 109 Cp/Cpk analysis Histogram Control chart Check sheet

d

You have been given the following sample information that has been drawn from the last sample taken: Mean of the sample: 12.50 Lower control limit for the range: 0 Upper control limit for the range: 10.6 Observations: 13, 12, 8, 10, 11, 16, 17, 8, 17, 18, 20, 14 Which of the following conclusions does this data support? The process is under control The process is not under control. The sample is within specifications The sample is out of specification

d

Types of Supply Chain Operational Technologies

decision support systems, processing technologies, communications technologies, interactive technologies

Which two of the following effects directly result from bottlenecks?

decrease in output and increase in lead-times

You walk into a hair stylist shop. All stylists are busy and you sit in the waiting area. You are in which of the following basic activity types in a process?

delay

Which of the following statements about economies of scale is NOT true?

economies of scale refers to the fact that as volume increases, total cost of production decreases

When a process is automated:

fixed costs increase

when a process is automated:

fixed costs increase

The total time that it takes to get one unit through a process is called:

flow time

All of the following are outputs from an MRP system EXCEPT:

forecasts

full partnerships

have close working relations, trust, mutual respect, and highly integrated operations. acknowledge their interdependencies and work together to reduce total costs so both parties benefit.

mass service

high labor intensity, low customization/customer interaction (e.g. retail banks and gas stations)

Process thinking causes managers to address critical process elements, including:

if you dont like the outcome, change the process

A difference between periodic review and continuous review inventory systems is:

in one system time triggers orders, in the other quantity triggers orders

A difference between periodic review and continuous review inventory systems is:

in one system, time triggers orders; in other, quantity triggers orders

Floor space used to store items to be inspected belongs to which of the following elements of a process?

inputs/outputs/flows

Littles Law: I/TH

inventory/ throughput

Supplier Relationship Management (SRM)

is a comprehensive system, facilitated by software, that manages the firm's interactions with its supply base. The goal of it is to streamline the processes and interactions that exist between the firm and its various suppliers so they are more efficient and transparent.

Supplier certification

is an assessment that verifies that the supplier operates, maintains, improves, and documents effective procedures related to the buyer's requirements.

If the planning horizon is 12 weeks, the cumulative lead time must be:

less than or equal to 12 weeks

An end item A is made by assembling four of component B and one of component C. Component B is made from two of part D and four of part E. Within the bill of materials, component B is considered to be:

level 1

weighted-point model

links the supplier's performance rating to the firm's competitive priorities.

A bill of materials file contains:

lists of materials required to produce end item products

service shop

low labor intensity, high customization/customer interaction (e.g. repair shops and hospitals)

service factory

low labor intensity, low customization/customer interaction (e.g. trucking companies, airlines, and hotels)

Zanda Corp. wants to have a process structure that has cost advantages similar to continuous or repetitive processes, but wants to produce greater variety than those processes normally allow. Zanda should consider:

mass customization

Using technology such as self-checkouts in grocery stores to enable customers to complete the service delivery and transaction themselves is most commonly used for"

mass services

Using technology such as self-checkouts in grocery stores to enable customers to complete the service delivery and transaction themselves is most commonly used for:

mass services

back-office/invisible contact employee actions

nonvisible interactions with customers, such as telephone calls, as well as other activities employees undertake to prepare to serve customers

Which of the following is NOT an input into the MRP?

planned order release

The entire time period covered by the MPS is the:

planning horizon

Zanadu Corp. has dedicated equipment and workers in a regularly occurring sequence of activities. Zanadu has which type of operations layout?

product layout

assemble to order (ATO)

products that are produced from standard components and modules (e.g. paint stores mixing coloring agents with a white base paint after an order is placed)

make to order (MTO)

products that have similar designs but are customized during production (e.g. a trip to the emergency room and a haircut)

Types of Processes

project, job shop, batch, repetitive process, and continuous process

decision support systems

provide computing power and data management to make higher-quality decisions faster (e.g. warehouse management system)

Companies with a make to stock orientation are most likely to use which of the following process structures?

repetitive process

Strategic purchases

represent a high spend level and are high risk. Typically these purchases are unique and core to the firm's performance.

adversarial relationships

represent the traditional way that buyers and suppliers have approached each other. These relationships are typified by distrust, limited communications, and short-term business transactions.

Acceptance of mutual goals

represents a major step toward collaboration, but it lacks the commitment of resources associated with full partnership.

Little's Law suggests that the key to increased throughput is

shorter flow times

Little's Law suggests that the key to increased throughput is:

shorter flow times

The following sequence shows four operations for a computer chip assembly process and the effective capacity of each. Which step is the bottleneck? Step 1: 500 chips/hour Step 2: 250 chips/hour Step 3: 200 chips/hour Step 4: 550 chips/hour

step 3

Arm's-length relationships

tend to be limited to simple purchasing transactions, but they lack the high levels of distrust and antagonism often associated with adversarial relationships

A company has decided to outsource the production of key components for its main product to suppliers. What effect will this have on the product's bill of materials?

the bill of materials will be flatter

supply base optimization

the determination of the number of suppliers to use

fixed-position layout

the layout used when the product cannot be moved during production

A bottleneck activity in a process is generally the activity with:

the least capacity

takt time

the maximum allowable cycle time at each workstation based on customer demand (total takt time/takt time, round up)

supply chain risk

the probability of an unplanned event in acquisition, delivery, and use that negatively affects a firm's ability to serve its customers.

cellular manufacturing

the production of products with similar process characteristics on small assembly lines called cells; cells can make job shops or batch processes more efficient, it can increase the flexibility of repetitive processes, and processing time, inventory, material flow distance, and setup times are reduced

cycle time

the time that it takes to process one unit at an operation in the overall process

3D printing (additive manufacturing)

this process makes products by putting down successive layers of thin materials such as plastic, metal, ceramics, or food; it eliminates the need for producing parts and assembly by printing products in one piece, it can be made close to the customer (reducing inventory and transportation), it uses relatively low-cost equipment, and is highly flexible

Safety stock exists for which of the following reasons?

to provide protection against the uncertainties of supply and demand

engineer to order (ETO)

unique, customized products (e.g. a custom-build house, a cruise ship, and specialized industrial equipment)

mass customization

uses advanced technologies to customize products quickly and at a low cost


Related study sets

8 Steps of Policy Making Intro to Public Policy

View Set

MUSICOL 121: Chapter 17 - Chopin's Piano

View Set

HA CoursePoint: Children & Adolescents

View Set

Lifting and Moving Patients : Chapter 3

View Set

Principles of Microeconomics Exam 1

View Set

CCNA 2 v7.0 Final - What Protocol

View Set